Problemas Estadisticas

March 12, 2017 | Author: Erick Santiago Guillaumin Perez | Category: N/A
Share Embed Donate


Short Description

Download Problemas Estadisticas...

Description

C´alculo de Probabilidades. Enunciados.

25 de septiembre de 2006

2

´Indice general 1. Espacio de probabilidad 1.1. Probabilidad . . . . . . . . . . . . . . . . . . . . . . . . . . . . . . . . . . . . . . 1.2. Probabilidad condicionada, teorema de Bayes e independencia . . . . . . . . . . . 2. Variables y vectores aleatorios 2.1. Variable aleatoria . . . . . . . . . . . . . . . 2.2. Vector aleatorio . . . . . . . . . . . . . . . . 2.3. Independencia de variables aleatorias . . . . 2.4. Distribuciones condicionadas . . . . . . . . 2.5. Funci´ on de una o varias variables aleatorias

. . . . .

. . . . .

. . . . .

. . . . .

. . . . .

. . . . .

. . . . .

. . . . .

. . . . .

. . . . .

. . . . .

. . . . .

. . . . .

. . . . .

. . . . .

. . . . .

. . . . .

. . . . .

. . . . .

. . . . .

. . . . .

5 5 8 17 17 21 21 23 24

3. Esperanza 27 3.1. Esperanza de una variable aleatoria . . . . . . . . . . . . . . . . . . . . . . . . . . 27 3.2. Esperanza de un vector aleatorio . . . . . . . . . . . . . . . . . . . . . . . . . . . 32 3.3. Esperanza condicionada . . . . . . . . . . . . . . . . . . . . . . . . . . . . . . . . 35 4. Convergencia de sucesiones de variables 4.1. Tipos de convergencia . . . . . . . . . . 4.2. Leyes de los Grandes N´ umeros . . . . . 4.3. Funci´ on caracter´ıstica . . . . . . . . . . 4.4. Teorema Central de L´ımite . . . . . . . 4.5. Funci´ on generatriz de momentos . . . .

aleatorias . . . . . . . . . . . . . . . . . . . . . . . . . . . . . . . . . . .

. . . . .

. . . . .

. . . . .

. . . . .

. . . . .

. . . . .

. . . . .

. . . . .

. . . . .

. . . . .

. . . . .

. . . . .

. . . . .

. . . . .

. . . . .

. . . . .

39 39 40 41 41 42

5. Ex´ amenes previos 5.1. 1 de septiembre de 2004 5.1.1. Castellano . . . . 5.1.2. Valenciano . . . 5.2. 3 de febrero de 2004 . . 5.2.1. Castellano . . . . 5.2.2. Valenciano . . . 5.3. 3 de septiembre de 2005 5.3.1. Castellano . . . . 5.3.2. Valenciano . . . 5.4. 8 de junio de 2004 . . . 5.4.1. Castellano . . . . 5.4.2. Valenciano . . . 5.5. 9 de febrero de 2005 . . 5.5.1. Castellano . . . .

. . . . . . . . . . . . . .

. . . . . . . . . . . . . .

. . . . . . . . . . . . . .

. . . . . . . . . . . . . .

. . . . . . . . . . . . . .

. . . . . . . . . . . . . .

. . . . . . . . . . . . . .

. . . . . . . . . . . . . .

. . . . . . . . . . . . . .

. . . . . . . . . . . . . .

. . . . . . . . . . . . . .

. . . . . . . . . . . . . .

. . . . . . . . . . . . . .

. . . . . . . . . . . . . .

. . . . . . . . . . . . . .

. . . . . . . . . . . . . .

. . . . . . . . . . . . . .

43 43 43 44 45 45 46 47 47 47 48 48 49 50 50

. . . . . . . . . . . . . .

. . . . . . . . . . . . . .

. . . . . . . . . . . . . .

. . . . . . . . . . . . . .

. . . . . . . . . . . . . .

. . . . . . . . . . . . . .

. . . . . . . . . . . . . .

. . . . . . . . . . . . . .

3

. . . . . . . . . . . . . .

. . . . . . . . . . . . . .

. . . . . . . . . . . . . .

. . . . . . . . . . . . . .

. . . . . . . . . . . . . .

. . . . . . . . . . . . . .

. . . . . . . . . . . . . .

´INDICE GENERAL

4

5.5.2. Valenciano 5.6. 21 de junio de 2005 5.6.1. Castellano . 5.6.2. Valenciano 5.7. 6 de junio de 2006 5.7.1. Castellano . 5.7.2. Valenciano

. . . . . . .

. . . . . . .

. . . . . . .

. . . . . . .

. . . . . . .

. . . . . . .

. . . . . . .

. . . . . . .

. . . . . . .

. . . . . . .

. . . . . . .

. . . . . . .

. . . . . . .

. . . . . . .

. . . . . . .

. . . . . . .

. . . . . . .

. . . . . . .

. . . . . . .

. . . . . . .

. . . . . . .

. . . . . . .

. . . . . . .

. . . . . . .

. . . . . . .

. . . . . . .

. . . . . . .

. . . . . . .

. . . . . . .

. . . . . . .

. . . . . . .

. . . . . . .

. . . . . . .

. . . . . . .

. . . . . . .

51 51 51 52 54 54 55

Cap´ıtulo 1

Espacio de probabilidad 1.1.

Probabilidad

Problema 1 Juego de dados tradicional chino que se juega durante la celebraci´ on del a˜ no nuevo. En este juego se lanzan 6 dados. Seg´ un parece un lanzamiento con dos pares gana a un lanzamiento con un par. ¿Cu´ al es la probabilidad de cada uno de estos sucesos? En otras palabras, encuentra la probabilidad de obtener un par en un lanzamiento de 6 dados y la probabilidad de obtener dos pares en un lanzamiento de 6 dados. Problema 2 (Problema de los cumplea˜ nos) En una reuni´ on hay n personas. ¿Cu´ al es la probabilidad de que dos de ellas tengan el mismo cumplea˜ nos? Problema 3 (Pitman, p´ agina 9) Elegimos una palabra al azar de esta frase. Se pide: 1. ¿Qu´e probabilidad tenemos de que la palabra tenga al menos cuatro letras? 2. ¿Y de que la palabra tenga al menos dos vocales? 3. ¿Y de que tenga al menos dos letras y al menos dos vocales? Problema 4 (Muestreo con y sin reemplazamiento) Veamos un experimento que corresponde a lo que se conoce como muestreo con reemplazamiento Una caja contiene una serie de papeletas marcadas con los n´ umeros 1, . . . , n. Elegimos al azar una papeleta de la caja. Vemos su n´ umero y la devolvemos a la caja. Determinar las probabilidades de los siguientes sucesos. 1. La primera papeleta tiene el n´ umero 1 y la segunda el n´ umero 2. 2. Los n´ umeros de las dos papeletas son n´ umeros enteros consecutivos, esto es, la primera papeleta tiene un n´ umero una unidad inferior a la segunda. 3. El segundo n´ umero extraido es mayor que el primero. Supongamos ahora que no reemplazamos la primera papeleta en la caja. En consecuencia la segunda papeleta ha de ser distinta a la primera. Se pide responder a las tres preguntas anteriores en esta nueva situaci´ on. Problema 5 Supongamos que barajamos una baraja de 52 cartas y tomamos las dos cartas que han quedado en la parte superior del mazo.

5

CAP´ITULO 1. ESPACIO DE PROBABILIDAD

6

1. ¿Cu´ antos pares ordenados de cartas podemos obtener como resultado? En lo que sigue vamos a asumir que cada uno de estos pares tiene la misma probabilidad de producirse. 2. ¿Qu´e probabilidad tenemos de que la primera carta sea un as? 3. ¿Qu´e probabilidad tenemos de que la segunda carta sea un as? 4. ¿Y de que ambas cartas sean ases? 5. ¿Y de que al menos tengamos un as entre las dos cartas? Problema 6 Tenemos diez puntos colocados de forma equidistante en la circunferencia de un c´ırculo y elegimos aleatoriamente tres de entre esos diez puntos. Se pide: 1. Si A y B son dos puntos particulares adyancentes, ¿qu´e probabilidad tenemos de que A y B est´en entre los puntos seleccionados? 2. ¿Qu´e probabilidad tenemos de que entre los tres puntos seleccionados aleatoriamente tengamos como m´ınimo un par de puntos adyacentes? Problema 7 Se dispone de n1 cubos blancos y n2 cubos rojos. Se los ordena al azar en compartimentos numerados de 1 a n1 + n2 . 1. ¿Cu´ al es el n´ umero N de disposiciones distintas posibles? 2. Calcular la probabilidad de que K cubos blancos determinados se encuentren en lugares fijados. Problema 8 (Examen 9-2-2005) ¿Cu´ al es la probabilidad de que una mano de p´ oquer contenga s´ olo una pareja? Nota: una baraja de p´ oquer tiene cuatro palos y de cada palo hay 13 cartas. En una mano se sirven cinco cartas. Problema 9 (Poker) La baraja francesa consta de 52 cartas distribuidas en cuatro palos o colores: tr´eboles, diamantes, corazones y picas. Cada uno de estos palos est´ a compuesto por 13 cartas: uno o as, dos, tres, cuatro, cinco, seis, siete, ocho, nueve, diez y las tres figuras, que se llaman valet (V, equivalente al Bube alem´ an, al Jack ingl´es, e incluso puede asimilarse a la Sota espa˜ nola), Dame (D, equivalente a la Dame alemana y a la Queen inglesa) y Roi (R, equivalente al K¨ onig alem´ an, al King ingl´es, y tambi´en al Rey de la baraja espa˜ nola). En una mano de poker se reparten 5 cartas a cada jugador. Se pide hallar la probabilidad de cada uno de estos sucesos: 1. Tener escalera de color (5 cartas consecutivas del mismo palo). 2. Tener poker (4 cartas iguales x x x x y). 3. Tener un full (un tr´ıo y una pareja x x x y y). 4. Tener 5 cartas del mismo palo. 5. Tener una escalera (5 cartas consecutivas). 6. Tener un tr´ıo (x x x y z) 7. Tener dobles parejas (x x y y z).

1.1. PROBABILIDAD

7

8. Tener una pareja (x x y z w). Problema 10 Lanzamos dos dados. Determinar la probabilidad de los siguientes sucesos: 1. El m´ aximo de los dos valores que obtenemos es menor o igual a 2. 2. El m´ aximo de los dos valores es menor o igual a 3. 3. El m´ aximo de los dos n´ umeros es igual a 3. 4. Repite los dos apartados anteriores sustituyendo 3 por x donde x var´ıa entre 1 y 6. 5. Si denotamos por p(x) con x = 1, . . . , 6 las probabilidades calculadas en el apartado anteP6 rior comprueba que i=1 p(x) = 1.

Problema 11 (Una carrera de tortugas) En la carrera de las grandes tortugas compiten cuatro animales. Para darle un poco de animaci´ on a la carrera, los cuatro propietarios deciden introducir los nombres de las tortugas en un sombrero y cada uno de los propietarios elige aleatoriamente un nombre sin reemplazamiento. Cada propietario est´ a obligado a apostar por la tortuga que le ha correspondido en el sorteo. Se pide: 1. Determinar la probabilidad de que todos los propietarios apuesten por sus tortugas. 2. Ning´ un propietario apueste por su propia tortuga. 3. El desafortunado propietario A elija a la tortuga Berzine que siempre pierde. 4. A apueste por la tortuga de B y B por la tortuga de A.

Problema 12 (Examen 2-3-2004) El holand´es Christian Huygens public´ o en 1657 uno de primeros libros sobre Probabilidad que se conocen, De Ratiociniis in Ludo Aleae (Del Razonamiento en los Juegos de Azar), en el que planteaba una serie de problemas. El que se conoce como segundo problema de Huygens lo enunciamos a continuaci´ on Tres jugadores A, B y C participan en el siguiente juego. Una urna contiene a bolas blancas y b negras. Los jugadores, en el orden ABCABC . . ., extraen una bola con reemplazamiento hasta que uno de ellos obtiene una bola blanca y gana. Encontrar la probabilidad de ganar para cada jugador. Problema 13 Hemos cuadriculado una cierta zona en seis columnas y cuatro filas. Denotamos por C(i, j) la celda en la fila i y columna j. Considerad el siguiente juego. Tenemos una ficha colocada en el rect´ angulo marcado con C(0, 0). La ficha la movemos a la derecha o hacia arriba desde la celda inicial C(4, 1) a la celda final C(1, 6). Se pregunta: 1. ¿Cu´ antos posibles caminos hay moverse desde C(4, 1) hasta C(1, 6)? 2. Si el jugador en su camino pasa por la celda C(2, 5) recibe un premio. Supongamos que cada camino tiene la misma probabilidad, ¿cu´ al es la probabilidad de que el jugador pase por C(2, 5) en su camino de C(4, 1) a C(1, 6)? Problema 14 (Aditividad finita y numerable) Demostrar que una medida de probabilidad es finitamente aditiva. Problema 15 Comprobar que la definici´ on de probabilidad de Laplace verifica los axiomas de Kolmogorov. En definitiva, que es una medida de probabilidad.

CAP´ITULO 1. ESPACIO DE PROBABILIDAD

8

Problema 16 (Krief y Levy, p´ agina 81) Tenemos el espacio de probabilidad (Ω, A, P ). Se pide: 1. Probar que si A, B y C son tres sucesos en este espacio se tiene que: P (A ∪ B ∪ C) = P (A) + P (B) + P (C) − P (A ∩ B) − P (B ∩ C) − P (A ∩ C) + P (A ∩ B ∩ C). 2. Sean A1 , . . . , An sucesos en (Ω, A, P ). Demostrar la desigualdad siguiente: P (A1 ∪ . . . ∪ An ) ≤

n X

P (Ai ).

i=1

¿En qu´e caso la desigualdad anterior es una igualdad? Problema 17 (Krief y Levy p´ agina 81) Sea (Ω, A, P ) un espacio de probabilidad. Se denomina diferencia sim´etrica de dos sucesos A y B al suceso A△B = (A ∩ B c ) ∪ (Ac ∩ B). 1. Probar que si tenemos los tres sucesos A, B y C entonces P (A△C) ≤ P (A△B) + P (B△C). 2. Para sucesos A, B, C y D se verifica   P (A ∪ B)△(C ∪ D) ≤ P (A△C) + P (B△D). Problema 18 (Un camino aleatorio cul´ e) El d´ıa 27 de julio de 1997 se celebraron elecciones a la presidencia del Bar¸ca. Hab´ıa s´ olo dos candidatos, el se˜ nor Fern´ andez y el se˜ nor N´ un ˜ez, siendo este u ´ltimo el ganador. Un socio con veleidades probabil´ısticas se hizo la siguiente pregunta: ¿habr´ a ido el se˜ nor N´ un ˜ez por delante del se˜ nor Fern´ andez a lo largo de todo el escrutinio? El se˜ nor N´ un ˜ez obtuvo 24025 votos y el se˜ nor Fern´ andez 5209.

1.2.

Probabilidad condicionada, teorema de Bayes e independencia

Problema 19 Sean A y B dos sucesos. Obtener la probabilidad de A ∩ B si ha ocurrido A o si ha ocurrido A ∪ B. Comentar el resultado. Problema 20 (La paradoja del caballero De Mer´ e) En un juego consistente en lanzar repetidamente un par de dados, encontrar el menor n´ umero n de lanzamientos para que la probabilidad de obtener al menos un doble seis sea mayor que 0,5. Comentarios El origen de la paradoja est´ a en la pregunta que Antoine Gombauld, caballero De Mer´e, plante´o a Pascal Observaba De Mer´e una discrepancia entre la realidad, deducida de su larga experiencia como jugador, y una antigua regla muy extendida entre los jugadores que afirmaba que n = 24. Esta err´ onea regla ten´ıa su origen en la creencia de un comportamiento lineal de las probabilidades. Se sab´ıa que si los lanzamientos eran de un solo dado y se persegu´ıa la obtenci´ on de un seis, n = 4, pues p3,1 = 0,4213 y p4,1 = 0,5177. Se razonaba a continuaci´ on mediante una sencilla regla de tres: 4 es a 6 como 24 a 36.

1.2. PROBABILIDAD CONDICIONADA, TEOREMA DE BAYES E INDEPENDENCIA 9

Problema 21 A y B juegan a un juego en el que A tiene una probabilidad p de ganar una partida. El vencedor es aquel que gana dos partidas consecutivas. Encontrar el valor de p si se sabe que cuando A pierde la primera partida, las probabilidades de ganar el juego para A y para B son iguales. Problema 22 (Jugando con un tetraedro) Tenemos un tetraedro un poco extra˜ no. Una cara es de color rojo, la otra de color azul y una tercera de color verde. La cuarta y u ´ltima cara tiene tres partes coloreadas respectivamente de rojo, azul y verde. Lanzamos el tetraedro y nos fijamos en el color de la cara en que se apoya el tetraedro. Consideramos los sucesos Ar consistente en que se apoya en una cara con el color rojo. Definimos Aa y Av anal´ agomente sustituyendo el color rojo por el azul y el verde. ¿Son independientes dos a dos los sucesos aleatorios Ar , Aa y Av ? ¿Son independientes los tres sucesos? Problema 23 Con objeto de estudiar la efectividad de un nuevo remedio contra el dolor de cabeza una gran muestra formada por n personas ha sido seleccionada. Puesto que se piensa que la reacci´ on de una persona al medicamento puede estar relacionada con el sexo, los datos que se tomaron fueron espec´ıficos al sexo. La siguiente tabla resume los resultados obtenidos: H M

E a c

Ec b d

donde E = { el medicamento ha sido efectivo }, E c = { el medicamento no ha sido efectivo }, H = { hombre }, M = { mujer } y a + b + c + d = n. Es decir, el n´ umero de mujeres para las cuales ha sido efectivo el medicamento es c y as´ı con el resto de la tabla. 1. Bas´ andose en estos datos, determinar (i) P(H), (ii) P(E), (iii) P(H | E), (iv) P(E | H). 2. ¿Cuando son independientes los sucesos E y H? Demostrar que si ad − bc = 0, E y M son sucesos independientes. 3. Si E y M son independientes, decimos que el efecto medicamento y el factor sexo son independientes. ¿Por qu´e podemos hacer esta afirmaci´ on? Problema 24 Demostrar que si B1 , . . . , Bn es una partici´ on de B (es decir, son disjuntos y ∪ni=1 Bi = B), entonces P(A | B) = P(A | B1 )P(B1 | B) + . . . + P(A | Bn )P(Bn | B). Problema 25 Consideremos un espacio de probabilidad (Ω, A, P ). Se pide: 1. Demostrar que si A y B son sucesos independientes tambi´en lo son A y B c ; Ac y B; Ac y Bc. 2. Tres sucesos A, B y C se dicen independientes si las tres parejas (A, B), (A, C) y (B, C) est´ an constituidas por sucesos independientes, y si P (A ∩ B ∩ C) = P (A) · P (B) · P (C). Se pide comprobar que las ternas (Ac , B, C), (Ac , B c , C) y (Ac , B c , C c ) son sucesos mutuamente independientes si A, B y C lo son. 3. Demostrar, con un ejemplo, que aunque las parejas (A, B), (A, C) y (B, C) est´en constituidas por sucesos independientes, no tiene porqu´e ocurrir lo mismo con la terna (A, B, C).

CAP´ITULO 1. ESPACIO DE PROBABILIDAD

10

Problema 26 (Krief y Levy, p´ agina 82) Se consideran cuatro n´ umeros reales a, b, c y d comprendidos entre 0 y 1. Determinar las condiciones necesarias y suficientes que deben verificar estos cuatro n´ umeros para que se pueda definir un espacio probabil´ıstico (Ω, A, P ) y dos sucesos A y B en este espacio verificando que: P (A|B) = a, P (A|B c ) = b, P (B|A) = c y P (B|Ac ) = d. Problema 27 Hay sucesos que son independientes de s´ı mismo. Por ejemplo, el suceso vac´ıo ∅ verifica que P (∅ ∩ ∅) = P (∅) = 0 y por lo tanto es independiente de s´ı mismo. ¿Qu´e ha de verificar un suceso para que sea independiente de s´ı mismo? Problema 28 Dos jugadores A y B juegan a un juego en el que cada uno de ellos puede efectuar n lanzamientos de dos dados, siendo A quien comienza. Las reglas del juego son las siguientes: Si A obtiene una suma 6 con los dados antes de que B haya obtenido una suma 7, A gana el juego. Si es B quien obtiene el 7 antes de que A haya obtenido el 6, es B quien gana. El juego termina en empate cuando ambos han agotado su n lanzamientos. Encontrar las expresiones de pA (n), pB (n) y pE (n) que denotan, respectivamente, que el ganador es A, el ganador es B o el juego termina en empate. Calcular sus l´ımites cuando n → ∞. Problema 29 Sean A y B dos sucesos incompatibles con probabilidad distinta de cero. ¿Cu´ al es la probabilidad de que A ocurra antes que B si el experimento se repite indefinidamente? Problema 30 (El juego de craps) Un jugador lanza dos dados, si la suma del primer lanzamiento es 7 u 11 gana, si la suma es 2, 3 o 12 pierde y si la suma es cualquier otro n´ umero continua lanzando hasta que aparezca una suma 7 o la suma que inicialmente obtuvo. Si aparece la suma 7 antes que la suma inicial pierde, en caso contrario gana. Calcular la probabilidad de que gane el juego. Problema 31 (El segundo problema de Huygens) El holand´es Christian Huygens public´ o en 1657 uno de primeros libros sobre Probabilidad que se conocen, De Ratiociniis in Ludo Aleae (Del Razonamiento en los Juegos de Azar), en el que planteaba una serie de problemas. El que se conoce como segundo problema de Huygens lo enunciamos a continuaci´ on Tres jugadores A, B y C participan en el siguiente juego. Una urna contiene a bolas blancas y b negras. Los jugadores, en el orden ABCABC . . ., extraen una bola con reemplazamiento hasta que uno de ellos obtiene una bola blanca y gana. Encontrar la probabilidad de ganar para cada jugador. Problema 32 (El problema de los puntos o del reparto de la apuesta) Dos jugadores A y B juegan a un juego consistente en un n´ umero indeterminado de partidas. La probabilidad de ganar en cada partida es p para A y 1 − p para B. Aquel de los dos que consigue antes vencer en r partidas gana el juego y la apuesta que ambos hicieron. Si el juego se interrumpe antes de finalizar, ¿c´ omo se debe repartir la apuesta? Problema 33 Utilizando argumentos probabil´ısticos, probar la igualdad (A > a) 1+

(A − a) . . . 2 · 1 A A − a (A − a)(A − a − 1) + + ··· = A−1 (A − 1)(A − 2) (A − 1) . . . (a + 1)a a

Sugerencia.- Una urna con A bolas de las cuales a son blancas, extracciones sucesivas sin reemplazamiento, primera bola blanca, etc.

1.2. PROBABILIDAD CONDICIONADA, TEOREMA DE BAYES E INDEPENDENCIA 11

Problema 34 ( ) Se lanza un dado una vez, si sale 1 se saca una bola de la urna I, si sale 2 o 3 se saca de la urna II, y en otro caso se saca de la urna III. La urna I tiene 5 bolas blancas, 3 verdes y 2 rojas; la urna II tiene 1 blanca 6 verdes y 3 rojas; la III tiene 3 blancas, 1 verde y 6 rojas. Determina las probabilidades siguientes: 1. que se elija una bola roja 2. que se haya seleccionado la urna II si ha salido roja. Problema 35 Proporcionamos a A un trozo de papel para que escriba un signo + o un signo −, sabiendo que escribe el primero con probabilidad 1/3. El papel pasa a B, quien lo deja como est´ a o cambia el signo antes de pasarlo a C. A continuaci´ on C, que puede o no haber cambiado el signo, lo pasa a D, quien finalmente nos lo devuelve tras haber introducido o no alg´ un nuevo cambio. Si comprobamos que el papel tiene escrito un signo + y sabemos que la probabilidad de que B, C y D cambiaran el signo es 2/3, obtener la probabilidad de que A escribiera originalmente un signo +. Problema 36 Un aparato de diagn´ ostico autom´ atico emite un diagn´ ostico basado en el resultado de n an´ alisis de un mismo paciente. Cada an´ alisis, independientemente de los restantes, puede dar un resultado err´ oneo con probabilidad p. La probabilidad de un buen diagn´ ostico, condicionada al n´ umero de an´ alisis correctos, es una funci´ on creciente de dicho n´ umero, g(m). Durante una ma˜ nana la m´ aquina ha diagnosticado a k pacientes. Encontrar la probabilidad del suceso A ={al menos un paciente est´ a mal diagnosticado}. Particularizar el resultado para g(m) = m/n. Problema 37 Un taxi se ve involucrado en un accidente nocturno. En la ciudad hay dos compa˜ n´ıas de taxis, los taxis Negros y los taxis Blancos. Se sabe que el 85 % de los taxis de la ciudad son Negros y el 15 % restante son Blancos. Un testigo del accidente afirma que el taxi involucrado era Blanco y la fiabilidad de su testimonio es del 80 %, es decir, es capaz de identificar correctamente el color del taxi el 80 % de las veces. 1. Sin ning´ un c´ alculo previo, ¿piensas que es m´ as probable que el taxi accidentado fuera el Negro o el Blanco? 2. Calcula la probabilidad de que el taxi accidentado fuera el Blanco y compara ambas respuestas. 3. Supongamos que para 0 ≤ p ≤ 1 el 100p % de los taxis son Blancos y que la fiabilidad del testigo contin´ ua siendo del 80 %. Estudia la sensibilidad a los datos de la respuesta anterior viendo como var´ıa ´esta en funci´ on de p. ¿A partir de qu´e valor de p la probabilidad de que haya sido el taxi Blanco el accidentado supera 0.5? 4. El an´ alisis anterior puede completarse permitiendo que la fiabilidad del testigo sea variable, 100q %, con 0 ≤ q ≤ 1. Determina la regi´ on dentro del cuadrado {(p, q) : 0 ≤ p ≤ 1, 0 ≤ q ≤ 1} en la que la probabilidad de que haya sido el taxi Blanco el accidentado supera 0.5. Cuando en todo cuanto precede nos referimos a la probabilidad de que haya sido el taxi Blanco se sobrentiende que dado que el testigo afirma que era Blanco.

12

CAP´ITULO 1. ESPACIO DE PROBABILIDAD

Problema 38 (El coche y las cabras) En un concurso de TV hay tres puertas, una de ellas esconde un coche y las otras dos sendas cabras. El concursante elige una de las puertas y obtiene como premio aquello que la puerta oculta, pero la puerta permanece cerrada y el presentador, que conoce lo que hay detr´ as de cada puerta, abre una de las otras dos puertas y aparece una cabra (l´ ogicamente el presentador nunca abre la puerta que oculta el coche). El presentador se dirige entonces al concursante y le permite cambiar su elecci´ on, ¿qu´e le conviene hacer al concursante? Este concurso tuvo gran ´exito a principios de los 90 en los USA y una conocida columnista de la revista Parade Magazine, Marylin vos Savant public´ o que cambiando su elecci´ on el concursante doblaba su probabilidad de ganar el coche, pues ´esta pasaba del 1/3 inicial a 2/3. Su afirmaci´ on era correcta. Compru´ebalo. Problema 39 El color de las flores de una cierta planta depende de dos genes, uno que recibe del padre y el otro de la madre. Si los dos genes son id´enticos entonces la flor tiene ese color; sin embargo, con genes diferentes la flor tiene bandas con cada uno de los dos colores. Los genes presentes en la poblaci´ on corresponden a los colores azul, amarillo y verde y su proporci´ on en la poblaci´ on es p, q y r (de modo que p + q + r = 1). Seleccionamos los padres de una planta aleatoriamente dentro de la poblaci´ on y consideramos: el suceso A consistente en que las flores del hijo tienen el color azul; el suceso B consistente en que las flores tengan m´ as de un color. Se pide: 1. Determinar la probabilidad de los dos sucesos. 2. Demostrar que los dos sucesos son independientes si p =

2 3

y q = r = 16 .

3. ¿Son estos los u ´nicos valores de p, q y r que hacen a los sucesos A y B independientes? Problema 40 Un test para diagnosticar cierta enfermedad tiene una sensibilidad del 95 % y una especificidad del 99 %. Si la enfermedad en cuesti´ on tiene una prevalencia del 0.5 %, ¿cu´ al es el valor predictivo del test? Problema 41 Supongamos una clase con n estudiantes. Uno de ellos conoce una historia sobre Jesul´ın. Se la cuenta a uno de sus compa˜ neros elegido al azar. A su vez este segundo estudiante vuelve a contarla a otro compa˜ nero diferente del que se la ha contado elegido al azar. El rumor sigue propag´ andose de este modo. En cada ocasi´ on la persona cuenta la historia a otra del grupo elegida al azar excluyendo a la persona que le ha informado. ¿Cu´ al es la probabilidad de que la historia se cuente k veces sin que se la cuenten dos veces al mismo individuo? Sugerencia:definid los sucesos Ai con i = 1, . . . , k consistentes en que la historia no se repite la i-´esima vez que se cuenta. A partir de estos sucesos definir el suceso de inter´es. Problema 42 Un sistema compuesto por n componentes trabaja en paralelo si funciona cuando al menos una componente funciona. Si la componente i funciona independendientemente de las dem´ as con probabilidad pi para i = 1, . . . n , ¿cu´ al es la probabilidad de que el sistema funcione? Problema 43 Ha desaparecido un avi´ on y se cree que es igualmente probable que se encuentre en cualquiera de las regiones R1 , R2 o R3 . Sea 1−αi la probabilidad de que se encuentre el avi´ on mientras se busca en la regi´ on i (en la pr´ actica estas probabilidades dependen de las condiciones geogr´ aficas y del entorno de las regiones). ¿Cu´ al es la probabilidad de que el avi´ on se encuentre en la regi´ on i dado que la b´ usqueda en la regi´ on 1 ha sido infructuosa? Problema 44 Si se lanzan dos dados equilibrados

1.2. PROBABILIDAD CONDICIONADA, TEOREMA DE BAYES E INDEPENDENCIA 13

1. ¿cu´ al es la probabilidad de que al menos uno de ellos sea un 6 dado que los dos n´ umeros que han salido son diferentes? 2. ¿cu´ al es la probabilidad de que el primero de ellos sea un 6 sabiendo que la suma de los dos n´ umeros que han salido es i? H´ allala para todos los valores de i entre 2 y 12. Problema 45 Es el doble de probable desarrollar un embarazo ect´ opico para una embarazada fumadora que para una embarazada no fumadora. Si el 32 % de las mujeres en edad f´ertil son fumadoras, ¿qu´e porcentaje de mujeres con embarazos ect´ opicos son fumadoras? Problema 46 Supongamos que el tiempo (seco o lluvioso) ma˜ nana ser´ a el mismo que el de hoy con probabilidad p. Si el tiempo el 1 de enero es seco, demuestra que Pn , que es la probabilidad de que sea seco n d´ıas despu´es, satisface: Pn = (2p − 1)Pn−1 + (1 − p), n ≥ 1 P0 = 1. Demuestra que Pn =

1 1 + .(2p − 1)n 2 2

Problema 47 Tres prisioneros A, B y C son informados por su carcelero de que se ha elegido al azar a uno de ellos para ser ejecutado y que los otros dos van a ser liberados. El prisionero A le pide al carcelero que le diga en privado cu´ al de sus compa˜ neros va a ser liberado, asegur´ andole que no pasa nada porque le d´e esa informaci´ on puesto que ´el sabe que al menos uno de los otros dos quedar´ a libre. El carcelero no quiere contestar la pregunta porque dice que si A supiera cu´ al de sus dos compa˜ neros va a ser liberado entones su propia probabilidad de ser ejecutado subir´ıa de 31 a 12 porque entonces ´el ser´ıa uno de los dos que podr´ıa ser ejecutado. ¿Qu´e piensas del razonamiento del carcelero? Problema 48 A y B se enfrentan en duelo. Las reglas del duelo son que ambos tienen que recoger el arma y disparar al otro simult´ aneamente. Si uno o ambos resultan heridos, el duelo se acaba. Si ambos fallan repiten el proceso. Supongamos que los resultados de los disparos son independientes y que un disparo de A alcanza a B con probabilidad pA y que un disparo de B alcanza a A con probabilidad pB . ¿Cu´ al es 1. la probabilidad de que A no resulte herido; 2. la probabilidad de que ambos duelistas resulten heridos; 3. la probabilidad de que el duelo acabe despu´es de n rondas de disparos; 4. la probabilidad de que el duelo acabe despu´es de n rondas de disparos dado que A no ha sido herido; 5. la probabilidad de que el duelo acabe despu´es de n rondas de disparos dado que ambos duelistas han sido heridos? Problema 49 Supongamos que tenemos 10 monedas de manera que si se lanza la i-´esima moi neda sale care cara con probabilidad 10 para i = 1, . . . , 10. Cuando se selecciona aleatoriamente al azar una moneda y se lanza sale cara, ¿cu´ al es la probabilidad de que la moneda seleccionada fuese la quinta?

14

CAP´ITULO 1. ESPACIO DE PROBABILIDAD

Problema 50 Dos armarios en apariencia id´enticos tienen dos cajones. El armario A contiene una moneda de plata en cada caj´ on y el armario B tiene una moneda de plata en un caj´ on y una moneda de oro en el otro caj´ on. Se elige al azar un armario, se abre uno de los cajones y se encuentra una moneda de plata, ¿cu´ al es la probabilidad de que haya una moneda de oro en el otro caj´ on? Problema 51 Un modelo simplificado para el cambio del precio de una acci´ on en bolsa supone que cada d´ıa el precio de la acci´ on aumenta 1 unidad con probabilidad p o baja 1 unidad con probabilidad 1 − p. Los cambios en d´ıas diferentes se consideran independientes. 1. ¿Cu´ al es la probabilidad de que despu´es de dos d´ıas el precio sea el mismo? 2. ¿Cu´ al es la probabilidad de que despu´es de dos d´ıas el precio haya aumentado en 1 unidad? 3. Dado que al cabo de tres d´ıas el precio de la acci´ on ha aumentado en 1 unidad, ¿cu´ al es la probabilidad de que subiera el primer d´ıa? Problema 52 Una baraja de poker (52 cartas) se divide al azar en 4 montones de 13 cartas cada uno. Calcula la probabilidad de que cada mont´ on contenga exactamente un as. Ayuda Define los sucesos Ei , para i = 1, 2, 3, 4 como sigue y usa la regla de la multiplicaci´ on: E1 = {el as de picas est´ a en cualquiera de los montones} E2 = {el as de picas y el as de corazones est´ an en montones diferentes} E3 = {los ases de picas, corazones y diamantes est´ an en montones diferentes} E4 = {los 4 ases est´ an en montones diferentes} Problema 53 Hay 12 bolas en una urna. Tres jugadores A, B y C extraen suces´ıvamente una bola de la urna (primero A, despu´es B y a continuaci´ on C). El ganador es el primero que extrae una bola blanca. Halla las probabilidades de ganar para cada jugador si 1. Cada bola se reemplaza despu´es de su extracci´ on. 2. Las bolas extraidas no se reintroducen en la urna. Problema 54 La probabilidad de ganar en un lanzamiento de dados es p. A empieza y si falla le pasa los dados a B, que intenta ganar en su turno. Contin´ uan tirando los dados suces´ıvamente hasta que uno de ellos gana. ¿Cu´ ales son las probabilidades de ganar de cada uno de ellos ? ¿Y si fueran k jugadores? Problema 55 Se busca un paraguas que, con probabilidad p7 , se encuentra en cualquiera de los siete pisos de un inmueble. Se han explorado en vano los seis primeros pisos. ¿Cu´ al es la probabilidad de que el paraguas se encuentre en el s´eptimo piso? Problema 56 (Krief y Levy, p´ agina 87) Una urna contiene bolas blancas y bolas negras. Se efect´ ua una sucesi´ on de n extracciones en la urna. Supongamos que la probabilidad de que 1 la k-´esima bola sacada sea blanca, cuando las k − 1 bolas precedentes lo fueron, es igual a k+1 . Calcular la probabilidad de que las n primeras bolas sacadas sean blancas. Problema 57 Una bola marcada puede estar en una cualquiera de las dos urnas que tenemos disponibles, con probabilidades p y 1 − p, respectivamente. La probabilidad de extraer la bola de la urna en la que est´ a alojada es r (r 6= 1). ¿Cu´ al es la mejor forma de utilizar n extracciones con reemplazamiento, de cualquiera de las dos urnas, para que la probabilidad de extraer la bola sea m´ axima?

1.2. PROBABILIDAD CONDICIONADA, TEOREMA DE BAYES E INDEPENDENCIA 15

Problema 58 Disponemos de 3 cajas de 20 piezas cada una. El n´ umero de piezas que re´ unen las condiciones de calidad exigidas son, respectivamente, 20, 15 y 10. De una de las cajas elegida al azar se extrae una pieza que resulta ser buena. Se devuelve a la caja y se extrae una segunda pieza que tambi´en resulta buena. ¿Cu´ al es la probabilidad de que la caja elegida haya sido la tercera? Problema 59 Dos especies muy parecidas de champi˜ nones (especies I y II) son dif´ıciles de distinguir sin la ayuda de un microscopio. Un m´etodo de campo habitualmente utilizado consiste en observar la presencia o ausencia de un anillo en el champi˜ no ´n. El 90 % de los individuos de la especie I y el 20 % de los de la especie II tienen el anillo. Se sabe tambi´en que en la zona en que se est´ a trabajando el 70 % de los champi˜ nones son de la especie I. Se pide: 1. Supongamos que el recolector encuentra un champi˜ no ´n con un anillo y decide que es de la especie I. ¿Con qu´e probabilidad est´ a en lo cierto? 2. Si todos los champi˜ nones con anillo son clasificados como de la especie I y los que no lo tienen como de la especie II, ¿qu´e proporci´ on de champi˜ nones estar´ a correctamente clasificado? Problema 60 (Krief y Levy, p´ agina 85) Dos personas escriben al azar un n´ umero entero de dos cifras (comprendidas entre 10 y 99). 1. Se repite la experiencia n veces y se supone que los resultados son mutuamente independientes. ¿Qu´e probabilidad tenemos de que las dos personas escriban, una vez al menos, el mismo n´ umero? Denotemos esta probabilidad por p(n). 2. Calcular p(100). 3. ¿Cu´ antas veces har´ıa falta repetir la experiencia para que p(n) sea igual al 0,99? Problema 61 Un jugador dispone de nueve dados: dos dados de tipo A, tres dados de tipo B y cuatro dados de tipo C. La tabla siguiente indica, para cada tipo de dado, el n´ umero de caras que llevan el n´ umero i (con i = 1, . . . , 6).

A B C

1 2 2 2

2 1 2 2

3 0 0 0

4 1 1 2

5 1 0 0

6 1 1 0

El jugador elige al azar uno solo de estos dados y hace 421 en tres tiradas. ¿Cu´ ales son las probabilidades respectivas de que haya jugado con un dado del tipo A, del tipo B o del tipo C? Problema 62 Se dispone de dos dados A y B. El dado A tiene cuatro caras rojas y dos caras blancas. El dado B tiene dos caras rojas y cuatro caras blancas. Se lanza una moneda: si se obtiene cruz se decide jugar u ´nicamente con el dado A; si se obtiene cara se decide jugar u ´nicamente con el dado B. Se pide calcular: 1. La probabilidad de obtener roja. 2. La probabilidad de obtener roja la tercera tirada sabiendo que ya se ha obtenido ese color en las dos primeras tiradas. 3. La probabilidad pn de haber utilizado el dado A sabiendo que se ha obtenido roja en las n primeras tiradas.

16

CAP´ITULO 1. ESPACIO DE PROBABILIDAD

Problema 63 (El problema de las coincidencias) Supongamos que 4 invitados llegan a una casa y dejan el sombrero en el vest´ıbulo. Si a la salida los recuperan de modo aleatorio, calcular la probabilidad de que ninguno de ellos reciba su propio sombrero. Resolver el mismo problema suponiendo que en lugar de cuatro invitados tenemos n invitados. Problema 64 Repetimos indefinidamente una prueba en la que la probabilidad de ´exito es siempre la misma, p, siendo los resultados de las pruebas independientes unos de otros (se trata de una sucesi´ on de pruebas de Bernoulli). Obtener la probabilidad de que a ´exitos ocurran antes que b fracasos. Problema 65 (La paradoja de la urna vac´ıa) Disponemos de una urna infinitamente grande y de una colecci´ on infinita de bolas numeradas. Procedemos a depositar las bolas en la urna de tres formas distintas. 1. A las 5 de la tarde menos 1 minuto introducimos las 10 primeras extrayendo la que lleva el n´ umero 10 (supongamos que la introducci´ on y la sucesiva extracci´ on consumen un tiempo 0). A las 5 menos 21 minuto depositamos las 10 bolas siguientes y extraemos la que lleva el n´ umero 20. A las 5 menos 41 las 10 siguientes extrayendo a continuaci´ on la que lleva el n´ umero 30. Y as´ı sucesivamente. 2. El segundo procedimiento es an´ alogo al anterior, pero las bolas que se extraen en cada en ocasi´ on son las numeradas 1, 2, 3, ..... 3. En el tercer procedimiento las bolas se introducen como en los dos anteriores pero en cada decena la extracci´ on se efectua al azar. ¿Cuantas bolas habr´ a en la urna a las 5 de la tarde seg´ un el procedimiento empleado? Problema 66 (Krief y Levy, p´ agina 88) Se considera un conjunto de N + 1 urnas numeradas. Cada urna contiene N bolas rojas o blancas. En concreto la urna k contiene k − 1 bolas blancas y N − k + 1 bolas rojas. Se escoge al azar una urna y se toman n bolas devolviendo a la urna cada bola extra´ıda antes de sacar la siguiente. 1. Determinar la probabilidad de que todas las bolas extra´ıdas sean blancas. 2. Siempre en la hip´ otesis de extracciones con reeemplazamiento, determinar la probabilidad de que la n + 1-´esima bola extra´ıda sea blanca sabiendo que las n bolas extra´ıdas anteriormente han sido blancas. Dar valores aproximados de estas probabilidades en el caso en que N es grande.

Cap´ıtulo 2

Variables y vectores aleatorios 2.1.

Variable aleatoria

Problema 67 Los autobuses llegan a la estaci´ on de Salat a intervalos de 10 minutos empezando desde las 12 : 00. Un hombre llega a la parada un n´ umero aleatorio de minutos X despu´es de las 12 : 00 si la funci´ on de distribuci´ on de X es:   0 si x < 0 x si 0 ≤ x ≤ 60 F (x) =  60 1 si x > 60 ¿Cu´ al es la probabilidad de que espere menos de 5 minutos?

Problema 68 Comprueba que la funci´ on fY (y) = 12 si y ∈ (−1, 1) (0 en otro caso) es una densidad de probabilidad de una variable aleatoria Y . Halla la funci´ on de distribuci´ on de dicha variable aleatoria. Problema 69 La funci´ on de probabilidad de una variable aleatoria X viene dada por fX (i) = λi c i! para i = 0, 1, . . ., donde λ es una constante positiva. Halla P (X = 0) y P (X > 2) Problema 70 Se eligen dos bolas al azar sin reemplazamiento de una urna que contiene 8 bolas blancas, 4 bolas negras y 2 de color naranja. Supongamos que ganamos 2 euros por cada bola blanca elegida y perdemos 1 euro por cada bola blanca elegida. Sea X la variable que denota nuestras ganancias. Halla la funci´ on de cuant´ıa (o probabilidad) de X. Problema 71 La funci´ on de distribuci´ on de X viene dada por:  0 si x < 0        x  si 0 ≤ x < 1  4     x−1 1 si 1 ≤ x < 2 F (x) = 2 + 4      11  si 2 ≤ x < 3  12       1 si x ≥ 3 1. Halla P (X = i) para i = 1, 2, 3

17

CAP´ITULO 2. VARIABLES Y VECTORES ALEATORIOS

18

2. Halla P ( 12 < X < 32 ) Problema 72 La funci´ on de distribuci´ on de X viene dada por:  0 si x < 0        1  si 0 ≤ x < 1  2         53 si 1 ≤ x < 2 F (x) =  4  si 2 ≤ x < 3  5       9  si 3 ≤ x < 3,5  10       1 si x ≥ 3,5

Halla la funci´ on de cuant´ıa de X.

Problema 73 Un vendedor de enciclopedias ha concertado dos citas con dos posibles compradores. Conseguir´ a vender una enciclopedia en la cita con el primer cliente con una probabilidad de 0,3 y lo lograr´ a en la cita con el segundo con una probabilidad de 0,6 (independientemente de lo que haya sucedido con el primero). Si vende la enciclopedia de lujo ingresa 1000 euros mientras que si vende la “normal” ingresa 500 euros. Supongamos que es igualmente probable que venda cualquiera de las dos enciclopedias. Halla la funci´ on de cuant´ıa de X que representa el total de los ingresos del vendedor. Problema 74 Cinco n´ umeros distintos se distribuyen al azar entre 5 jugadores numerados del 1 al 5. Cuando dos jugadores comparan sus n´ umeros el que lleva el n´ umero m´ as alto es el ganador. Inicialmente los jugadores 1 y 2 comparan sus n´ umeros, el ganador se compara con el jugador 3, y as´ı sucesivamente. Denotamos por X el n´ umero de veces que gana el jugador 1. Halla P (X = i) para i = 1, 2, 3, 4. Problema 75 Una moneda no correcta con probabilidad de cara p y probabilidad de cruz 1 − p es lanzada hasta que aparece una cara o tres veces, lo que ocurra antes. Si X es el n´ umero de lanzamientos que se realizan, se pide determinar la distribuci´ on de X. Problema 76 (Examen 9-2-2005) Probar que para cualquier funci´ on de densidad de probabilidad se verifica Z +∞ 1 l´ım x f (z)dz = 0. x→+∞ z x Problema 77 ¿Para qu´e valores de la constante C las funciones siguientes son funciones de cuant´ıa sobre los enteros positivos? 1. Geom´etrica f (x) = C2−x 2. Logar´ıtmica f (x) = C 2 x

−x

3. Inversa cuadrado f (x) =

C x2 x

4. Poisson modificada f (x) = C 2x!

19

2.1. VARIABLE ALEATORIA

Problema 78 El responsable de una tienda de electr´ onica compra cierta clase de piezas en lotes de tama˜ no 10. Su pol´ıtica consiste en inspeccionar 3 al azar de cada lote y aceptarlo s´ olo si las 3 funcionan correctamente. Si una quinta parte de los lotes contiene 4 piezas defectuosas y los dem´ as s´ olo una pieza defectuosa, ¿qu´e proporci´ on de lotes rechazar´ a? Problema 79 Para una distribuci´ on hipergeom´etrica halla H(N, n, r)?

P (X=k+1) P (X=k) .

¿Cu´ al es la moda de una

Problema 80 (Examen 9-2-2005) Una urna contiene n papeletas numeradas de 1 a n inclusive. Extraemos r al azar. Sea X el n´ umero mayor obtenido si las papeletas se reemplazan despu´es de cada extracci´ on y sea Y el n´ umero mayor si las papeletas no se reemplazan en la urna. Determinar las funciones de distribuci´ on, las funciones de cuant´ıa (o probabilidad) y demostrar que FY (k) < FX (k) para 0 < k < n. (2.1) Problema 81 En un proceso de fabricaci´ on de hilados se producen roturas del hilo de manera aleatoria a lo largo del tiempo. Es importante conocer cuando y c´ omo pueden producirse dichas roturas. Supongamos que un trabajador controla 800 husos y que la probabilidad de rotura del hilo en cada bobina, durante un cierto intervalo de tiempo τ , es p = 0,005. Encontrar el n´ umero de roturas m´ as probable y la probabilidad de que se produzcan a lo sumo 10 roturas. Problema 82 Samuel Pepy, contempor´ aneo de Isaac Newton, sab´ıa que al lanzar 6n dados el n´ umero esperado de seises era n. A partir de este resultado deduc´ıa que los sucesos An ={al menos n seises al lanzar 6n dados}, n = 1, 2, 3, ten´ıan todos igual probabilidad. Isaac Newton hubo de sacarlo de su error.1 Problema 83 Sea X el n´ umero de pruebas de Bernoulli necesarias para obtener un ´exito y un fracaso. Determinar la distribuci´ on de probabilidad de X. Problema 84 Una moneda de 1 cm de di´ ametro se lanza y cae dentro de una lata cil´ındrica cuyo fondo tiene 5 cm de di´ ametro (la moneda cae plana, no de canto). 1. ¿Cu´ al es la probabilidad de que la moneda cubra el centro del fondo de la lata? 2. Supongamos que en lugar de usar una lata cil´ındrica se tira en una caja cuyo fondo es un cuadrado cuyos lados miden 5 cm, ¿cu´ al es ahora la probabilidad de que la moneda cubra el centro del fondo de la lata? Problema 85 Un testigo experto en un juicio sobre una supuesta paternidad testifica que la longitud en d´ıas de un embarazo (es decir desde el momento de la concepci´ on hasta el momento del parto) se distribuye aproximadamente seg´ un una Normal con par´ ametros µ = 270 y σ = 10. El presunto padre puede demostrar que estuvo fuera del pa´ıs durante un per´ıodo de tiempo que empezaba 290 d´ıas antes del nacimiento del ni˜ no y que acababa 240 d´ıas antes del nacimiento. Si el acusado fuese realmente el padre de la criatura, y suponiendo que es verdad lo que asegura el experto ¿cu´ al ser´ıa la probabilidad de que la madre tuviera un embarazo tan largo o tan corto ? 1 El problema, que es de f´ acil soluci´ on y puede incluso parecer ingenuo a alg´ un lector, se recoge aqu´ı por su inter´ es hist´ orico y tambi´ en porque el autor de esta colecci´ on ha tenido ocasi´ on de comprobar que los ´ emulos actuales de Samuel Pepy son todav´ıa numerosos

CAP´ITULO 2. VARIABLES Y VECTORES ALEATORIOS

20

Problema 86 La mediana de una variable aleatoria cont´ınua con funci´ on de distribuci´ on F es aquel valor m tal que F (m) = 12 . Es decir es igual de probable que una variable aleatoria sea mayor que su mediana como que sea menor que ella. La moda de una variable aleatoria cont´ınua con funci´ on de densidad f es el valor de x para el que f (x) alcanza su m´ aximo. Halla la mediana y la moda de X si X se distribuye 1. U (a, b) 2. N (µ, σ) 3. Exp(λ) Problema 87 Un bit es trasmitido repiti´endolo n veces. El mensaje es interpretado asignando el valor que m´ as veces se recibe. Por ejemplo: si n = 5 y el mensaje recibido es 10010 entonces concluimos que se envi´ o un 0 (se repite 3 veces frente a las dos veces que se repite el 1). Suponiendo que n es un n´ umero impar y que cada bit del mensaje es transmitido correctamente con probabilidad p, independientemente de los dem´ as bits, determina la probabilidad de que el mensaje sea recibido correctamente (se reciba el bit que se trasmiti´ o). Problema 88 Consideremos la distribuci´ on Beta con par´ ametros a y b. Demuestra que 1. cuando a > 1 y b > 1, la densidad es unimodal, es decir tiene una u ´nica moda que es a−1 ; m = a+b−2 2. cuando a ≤ 1, b ≤ 1 y a + b < 2, la densidad es o bien unimodal con moda en 0 o en 1 o bien tiene forma de U con modas tanto en 0 como en 1; 3. cuando a = 1 = b, todos los puntos de [0, 1] son modas. Problema 89 Un fabricante de bolas para rodamientos somete su producto al siguiente proceso de control de calidad. Las bolas son aceptadas si no pasan a trav´es de un agujero de di´ ametro d1 , pero s´ı lo hacen a trav´es de otro de di´ ametro d2 , d2 > d1 . Se sabe que el di´ ametro D de las bolas es aleatorio con una distribuci´ on N (µ, σ 2 ), µ = (d1 + d2 )/2 y σ = (d2 − d1 )/4. ¿Cu´ al es la probabilidad de rechazar una bola? Problema 90 El tiempo que tardan en ser atendidos los clientes del servicio de caja de cierta sucursal bancaria es una variable aleatoria T ∼ Exp(λ), con λ = 0,2. Durante una ma˜ nana han llegado 10 clientes, ¿cu´ al es la probabilidad de que a lo sumo 3 de ellos hayan tardado m´ as de 6 minutos en ser atendidos? (Suponemos que los clientes son atendidos independientemente unos de otros). Problema 91 (Los sorteos de La Primitiva) Un asiduo de La Primitiva anda un tanto preocupado al comprobar que en los 18 u ´ltimos sorteos hay algunos n´ umeros que no han sido extra´ıdos. Piensa que las 108 extracciones (18 × 6) suponen un poco m´ as del doble de 49 y que por tanto cada n´ umero deber´ıa haber aparecido, aproximadamente, unas dos veces. ¿Y si el sorteo no fuera correcto? ¿Habr´ a n´ umeros m´ as probables que otros? Problema 92 Un comerciante vende semillas en paquetes de 50. Supongamos que cada semilla germina con una probabilidad de 0,99 independientemente de las dem´ as. El comerciante promete cambiar al comprador cualquier paquete que contenga 3 o m´ as semillas que no germinen. ¿Cu´ al es la probabilidad de que el comerciante tenga que cambiar m´ as de 40 paquetes de los 4000 que ha vendido?

21

2.2. VECTOR ALEATORIO

Problema 93 En una ciudad se va a someter a tratamiento a los ni˜ nos de seis a˜ nos m´ as bajos que cierta talla. En esta ciudad hay 6580 ni˜ nos de esta edad, y su estatura sigue una distribuci´ on normal de 119 cm de media y 4 cm de desviaci´ on t´ıpica. Se va a realizar una campa˜ na informativa indicando una talla por debajo de la cual se ha de tratar a los ni˜ nos. Los servicios de endocrinolog´ıa de la ciudad solo pueden atender a 750 ni˜ nos. Se pregunta: 1. ¿Qu´e talla en cm deber´ a indicar la campa˜ na? 2. ¿En cuantos ni˜ nos se ver´ a desbordado el servicio de endocrinolog´ıa si la campa˜ na indica por error un cm m´ as?

2.2.

Vector aleatorio

Problema 94 Lanzamos tres veces consecutivas una moneda y definimos las variables aleatorias X ={n´ umero de caras en los dos primeros lanzamientos} e Y ={n´ umero de caras en los dos u ´ ltimos lanzamientos}. Obtener la distribuci´ on de probabilidad conjunta de X e Y , sus marginales y el coeficiente de correlaci´ on entre ambas. Problema 95 Sean X1 , X2 , . . . , Xn variables aleatorias con funci´ on de distribuci´ on conjunta F y con funciones de distribuci´ on marginales F1 , F2 , . . . , Fn . Demostrar que 1−

n X i=1

[1 − Fi (xi )] ≤ F (x1 , x2 , . . . , xn ) ≤ m´ın Fi (xi ). 1≤i≤n

Problema 96 Los 4 tipos de sangre principales se presentan en la poblaci´ on de los EEUU de acuerdo con los siguientes porcentajes: Tipo Porcentaje

A 42 %

B 10 %

AB 4%

0 44

1. Si se eligen al azar a dos personas de esta poblaci´ on, ¿cu´ al es la probabilidad de que su sangre sea del mismo tipo? 2. Si se eligen cuatro personas al azar, sea P (k) la probabilidad de que haya exactamente k tipos sangu´ıneos diferentes. Halla P (k) para k = 1, 2, 3 y 4. Problema 97 Escogemos aleatoriamente un punto del interior de un disco de radio R. Sea X la distancia del punto elegido al centro del disco. Halla la funci´ on de distribuci´ on de X.

2.3.

Independencia de variables aleatorias

Problema 98 Consideremos el espacio de probabilidad (Ω, A, P ) y sean A1 y A2 dos sucesos. Definimos X1 = 1A1 y X2 = 1A2 , las funciones caracter´ısticas asociadas. Demostrar que X1 y X2 son independientes si y s´ olo si A1 y A2 lo son. Problema 99 Dos personas lanzan, cada una de ellas, n veces una moneda. Obtener la probabilidad de que ambas obtengan el mismo n´ umero de caras. Problema 100 Tenemos dos l´ıneas de comunicaci´ on telef´ onica paralelas de longitud l, separadas por una distancia d < l. Sabemos que, al azar y de manera independiente, se producen sendas roturas a lo largo del recorrido de cada una de ellas. Encontrar la probabilidad de que la distancia R entre ambas roturas no sea superior a c.

CAP´ITULO 2. VARIABLES Y VECTORES ALEATORIOS

22

Problema 101 Elegimos al azar dos puntos, X e Y , en el intervalo [0, a]. Calcular la funci´ on de distribuci´ on de la distancia entre ellos. Problema 102 Sea {Xk }k≥1 una colecci´ on de variables aleatorias independientes U(0,1). Sea 0 < x < 1, definimos N = m´ın{n ≥ 1 : X1 + X2 + · · · + Xn > x}. Encontrar P (N > n). Problema 103 Se escogen al azar dos n´ umeros a y b, a en el intervalo [1, 3] y b en el intervalo [−1, 1] . Halla la probabilidad de que la ecuaci´ on x2 + ax + b = 0 tenga dos ra´ıces reales. Problema 104 Escogemos dos n´ umeros al azar entre 0 y 1.¿Cu´ al es la probabilidad de que el primero sea mayor o igual que el cuadrado del segundo y al mismo tiempo que el segundo sea mayor o igual que el cuadrado del primero? Problema 105 Tenemos una urna con 12 bolas numeradas de 1 a 12. Extraemos dos bolas y denotamos por X1 y X2 los valores que observamos en la primera y en la segunda extracci´ on. Sea X la variable definida como el m´ aximo de las dos extracciones. Se pide: 1. Determinar la funci´ on de distribuci´ on de la variable X si suponemos que las dos extracciones se realizan con reemplazamiento. 2. Determinar la funci´ on de distribuci´ on de la variable X si suponemos que no hay reemplazamiento entre las dos extracciones. Problema 106 (Examen 11-2-2000) Las puntuaciones obtenidas por los estudiantes del centro A en las pruebas de selectividad se distribuyen N (µ = 6,25, σ 2 = 1), mientras que las de los estudiantes del centro se distribuyen N (6, 1,5). Elegimos al azar 2 estudiantes del centro A y 3 del centro B. Se pide: 1. la probabilidad de que la puntuaci´ on media de los 2 estudiantes de A sea superior a la puntuaci´ on media de los 3 de B, y 2. la probabilidad de que al escoger al azar uno de los 5 estudiantes, su nota sea superior a 6,5. Problema 107 Sean {Xk , k = 1, . . . , n} variables aleatorias i. i. d. con distribuci´ on U (0, 1). Tomemos 0 < x < 1 y definamos N = m´ın{n ≥ 1; X1 + X2 + · · · + Xn > x}. Encontrar P (N > n). Problema 108 Un grupo de 10 personas han quedado para comer entre las 12 : 00 y las 12 : 15. Cada persona llega al restaurante independientemente de las dem´ as y seg´ un una distribuci´ on uniforme en el intervalo de tiempo anterior. 1. Eva y Mar´ıa son dos miembros del grupo. Halla la probabilidad de que Eva llegue al menos dos minutos antes que Mar´ıa. 2. Halla la probabilidad de que la primera persona en llegar aparezca antes de las 12 : 05 y de que la u ´ltima llegue despu´es de las 12 : 10. Problema 109 Inyectamos a unos ratones microorganismos del tipo A y B en igual proporci´ on. Se supone que los microorganismos efectivos de cada tipo se distribuyen independientemente con arreglo a una misma distribuci´ on de Poisson de par´ ametro λ. Un rat´ on sobrevive si y s´ olo si no hay microorganismos efectivos en la dosis inyectada. A los ratones muertos se les examina para ver si conten´ıan microorganismos de uno o de los dos tipos. Encontrar la probabilidad de que un rat´ on muerto contenga microorganismos de un solo tipo.

2.4. DISTRIBUCIONES CONDICIONADAS

2.4.

23

Distribuciones condicionadas

Problema 110 Elegimos un n´ umero aleatorio X de la siguiente forma: lanzamos repetidamente una moneda hasta que nos muestre la primera cara, si el n´ umero lanzamientos que hemos necesitado es N , elegimos aleatoriamente un entero k en 1, 2, . . . , 10N , valor que asignamos a X. Encontrar la distribuci´ on de probabilidad de X. Problema 111 Sobre un c´ırculo cuyo radio R es aleatorio con funci´ on de densidad  2 r   , r ∈ [0, 3]; 9 fR (r) =   0 en el resto,

elegimos un punto al azar. Si X designa la distancia del punto al origen, obtener la funci´ on de distribuci´ on y la funci´ on de densidad de X|R = r. Problema 112 Consideremos la siguiente funci´ on  yx e−y x! , si y ≥ 0, x = 0, 1, . . . fX|Y (x|y) = 0, en el resto. a) Demostrar que para cada y fijo, fX|Y (x|y) es una funci´ on de probabilidad (la de X condicionada a Y = y). b) Si Y ∼ Exp(λ), con λ = 1, encontrar la densidad conjunta de X e Y . c) Comprobar que la marginal de X viene dada por  1  2x+1 , x = 0, 1, . . . fX (x) =  0, en el resto.

Problema 113 Las variables aleatorias X e Y tienen una distribuci´ on conjunta uniforme en el interior del tri´ angulo T de v´ertices (0,0), (2,0) y (1,2). Calcular P (X < 1, Y < 1) y P (Y < 1|X < 1,5). Problema 114 Calcular la probabilidad de poder formar un tri´ angulo con dos puntos elegidos en el intervalo [0, 1] seg´ un los distintos m´etodos que se enumeran a continuaci´ on. 1. Los dos puntos se eligen al azar en el intervalo. 2. Elegimos primero un punto al azar y a continuaci´ on elegimos el segundo punto, tambi´en al azar, pero sobre el trozo mayor. 3. Elegimos un punto al azar y a continuaci´ on uno de los trozos, elegido al azar, lo dividimos en dos partes iguales. Calcular, para este m´etodo, la probabilidad de que el tri´ angulo sea obtuso. Problema 115 (Examen 21-6-2005) En una urna hay una bola roja. Extraemos tres cartas de una baraja francesa (52 cartas repartidas en 4 palos) y a˜ nadimos a la urna tantas bolas verdes como ases hayamos extra´ıdo. A continuaci´ on lanzamos 2 veces una moneda cuya probabilidad de cara es p = 1/5 y a˜ nadimos tantas bolas rojas como cruces hayamos obtenido. Finalmente llevamos a cabo 2 extracciones con reemplazamiento de la urna. Si X es el n´ umero de bolas verdes a˜ nadidas a la urna e Y el n´ umero de bolas rojas a˜ nadidas a la urna,

CAP´ITULO 2. VARIABLES Y VECTORES ALEATORIOS

24

1. Obtener la funci´ on de probabilidad de X. 2. Obtener la funci´ on de probabilidad de Y . 3. Si las dos bolas extra´ıdas con reemplazamiento son rojas, ¿cu´ al es la probabilidad de no haber obtenido ning´ un as al extraer las 3 cartas de la baraja francesa?

2.5.

Funci´ on de una o varias variables aleatorias

Problema 116 Sea X una variable aleatoria cuya densidad viene dada por  0, si x < 0,      1 si 0 ≤ x ≤ 1, fX (x) = 2,      1 2x2 , si x > 1. Encontrar la densidad de Y = 1/X.

Problema 117 Se traza aleatoriamente una linea recta a trav´es del punto (0, l). Encontrar la densidad de probabilidad de la abcisa en el origen, X, de dicha recta. Problema 118 Sea λ el n´ umero de c´elulas por unidad de a ´rea en una preparaci´ on celular. Puede demostrarse que la distancia, D, de una c´elula a su vecino m´ as pr´ oximo tiene por densidad de probabilidad,  2 2πλde−λπd , d > 0 fD (d) = 0, en el resto. El a ´rea del mayor disco libre (sin contactos) centrado en una c´elula es A = πD2 . Encontrar la densidad de probabilidad de A. Problema 119 Las variables X e Y son independientes y continuas. Sea U una variable dicot´ omica, independiente de ambas con P (U = 1) = P (U = −1) = 1/2. Definimos S = U X y T = U Y . Comprobar que S y T son dependientes, pero S 2 y T 2 son independientes. Problema 120 Sean X e Y variables aleatorias independientes tales que X ∼ Gamma(α1 , β) e Y ∼ Gamma(α2 , β). Demostrar que las variables X + Y y X/(X + Y ) son variables aleatorias independientes. Problema 121 Tenemos X1 , . . . , Xn variables aleatorias independientes tales que Xj tiene una distribuci´ on binomial negativa con par´ ametros rj y p con j = 1, .P . . , n. Demostrar que Pn n Sn = j=1 Xj tiene una distribuci´ on binomial negativa con par´ ametros j=1 rj y p.

Problema 122 Tenemos X1 , . . . , Xn variables aleatorias independientes tales que Xj tiene una distribuci´ on geom´etrica con par´ ametro pj . Sea Nn = m´ın{X1 , . . . , Xn }. Demostrar que la variable Nn tiene una distribuci´ on geom´etrica con par´ ametro p=1−

n Y

j=1

(1 − pj ).

(2.2)

´ DE UNA O VARIAS VARIABLES ALEATORIAS 2.5. FUNCION

25

Problema 123 (Rohatgi, p´ agina 211) Sean X1 , X2 , . . . una sucesi´ on de variables aleatorias independientes e id´ e nticamente distribuidas (i.i.d.) con distribuci´ o n exponencial con par´ ametro Pn β. Sea Sn = k=1 Xk la n-´esima suma parcial, con n = 1, 2, . . . y supongamos t > 0. Si Y es el n´ umero de valores de Sn en el intervalo [0, t] entonces Y sigue una distribuci´ on de Poisson con par´ ametro t/β. Problema 124 (Examen 3-9-2005) Cuando una corriente de I amperios pasa a trav´es de una resistencia de R ohmios, la potencia generada viene dada por W = I 2 R vatios. Supongamos que I y R son variables aleatorias independientes con densidades   6x(1 − x), si 0 ≤ x ≤ 1; fI (x) =  0, fuera. y

fR (y) = Hallar la densidad de W .

  2y, si 0 ≤ y ≤ 1; 

0,

fuera.

Problema 125 Demostrar que si tenemos variables aleatorias Pn independientes X P1 ,n. . . , Xn tales que Xj ∼ Gamma(αj , β) con j = 1, . . . , n entonces Sn = j=1 Xj ∼ Gamma( j=1 αj , β).

Problema 126 (Rohatgi, p´ agina 213) Si X e Y son variables aleatorias exponenciales con par´ ametro β entonces Z = X/(X + Y ) sigue una distribuci´ on uniforme en el intervalo [0, 1].

Problema 127 (Rohatgi, p´ agina 215) Sean X e Y variables aleatorias independientes con distribuci´ on Gamma(α1 , β) y Gamma(α2 , β) respectivamente. Demostrar que X/(X + Y ) sigue una distribuci´ on beta con par´ ametros α1 y α2 . Problema 128 (Examen 8-6-2004) Consideremos el siguiente procedimiento: 1. Generamos U con distribuci´ on uniforme en [0, 1]. 2. Tomamos Y = − λ1 ln(1 − U ) siendo ln el logaritmo neperiano y λ una constante positiva. 3. Tomamos X = [Y ] donde [Y ] es la parte entera por exceso de Y . Se pide: 1. La distribuci´ on de probabilidad de la variable aleatoria Y . 2. La funci´ on de probabilidad de la variable X. Comprobar que tiene una distribuci´ on geom´etrica. Problema 129 Sean X, Y, Z variables aleatorias i. i. d. con distribuci´ on U (0, 1). Encontrar la densidad conjunta de XY y Z 2 y calcular P (XY ≤ Z 2 ).

26

CAP´ITULO 2. VARIABLES Y VECTORES ALEATORIOS

Cap´ıtulo 3

Esperanza 3.1.

Esperanza de una variable aleatoria

Problema 130 Dos jugadores A y B juegan al siguiente juego: lanzan una moneda hasta que aparece una cara. Si la cara aparece en el k-´esimo lanzamiento el jugador B paga al jugador A k monedas. ¿Cuantas monedas deber´ a pagar el jugador A antes de iniciarse el juego para que ´este sea equilibrado? Problema 131 En dos lados distintos de un cuadrado unidad elegimos al azar e independientemente los puntos X e Y . Si por D designamos la distancia entre ellos, calcular E(D2 ) en las dos situaciones posibles descritas en la figura.

Y D Y D

X

X Problema 132 Sean X e Y variables aleatorias independientes. Demostrar que si X y X − Y son independientes, entonces X es una variable aleatoria degenerada. Problema 133 Sea X una variable aleatoria discreta con soporte DX = {x1 , x2 , . . . , xm }. Calcular E(X n+1 ) . l´ım n→∞ E(X n ) Problema 134 Dos puntos se eligen al azar sobre un segmento de longitud a. Encontrar la esperanza y la varianza de la distancia entre ellos. Problema 135 (Si hay cumplea˜ nos no se trabaja) En algunos pa´ıses socialistas y en alg´ un momento de su historia se estableci´ o una ley laboral por la que en las factor´ıas e instituciones

27

CAP´ITULO 3. ESPERANZA

28

estatales se trabajaba todos los d´ıas de la semana. La excepci´ on a esta regla era que algunos de los trabajadores cumpliera a˜ nos, ese d´ıa nadie trabajaba. Suponiendo un a˜ no no bisiesto y que los cumplea˜ nos se distribuyen equiprobablemente a lo largo del a˜ no, encontrar el n´ umero de trabajadores para que el n´ umero esperado de hombres-d´ıa trabajados sea m´ aximo. Problema 136 Elegimos al azar un n´ umero del 1 al 10. Hemos de adivinarlo mediante preguntas cuya respuesta sea s´ı o no. Calcular el n´ umero esperado de preguntas que debemos hacer si, 1. la pregunta i-´esima que hacemos es ¿se trata del n´ umero i?, o 2. con cada pregunta intentamos eliminar la mitad de los n´ umeros restantes. Problema 137 Un vendedor de peri´ odicos compra cada ejemplar a 10 c´entimos y los vende a 15 c´entimos. Como no puede devolver los ejemplares no vendidos, trata de saber cuantos le conviene comprar para maximizar lo que espera ganar cada d´ıa. Si las demandas diarias siguen una distribuci´ on B(10, 1/3), ¿cu´ antos ejemplares debe comprar? Problema 138 Un vendedor de peri´ odicos compra cada ejemplar a C c´entimos de euro y lo vende a V c´entimos de euro (C < V ). Puede devolver los ejemplares no vendidos pero s´ olo obtiene R c´entimos de euro por ejemplar (R < C). Se trata de saber cuantos le conviene comprar para maximizar lo que espera ganar cada d´ıa. Si la demanda diaria X sigue una distribuci´ on discreta con P (X = k) = pk , k = 0, 1, 2, . . . , n, ¿cu´ antos ejemplares debe comprar? Aplicar el resultado para el caso en que V = 100, C = 50, R = 20 y la funci´ on de probabilidad viene dada por k 0 1 2 3 4 5 6

k 7 8 9 10 11 12 13

P(X=k) 0.05 0.08 0.10 0.15 0.15 0.10 0.08

P(X=k) 0.08 0.05 0.04 0.03 0.03 0.03 0.03

Problema 139 Si X es una variable aleatoria continua con media 0 y varianza σ 2 y funci´ on de distribuci´ on F , comprobar que se verifica F (x) ≥

x2

x2 , + σ2

si x > 0

y

F (x) ≤

x2

σ2 , + σ2

si x < 0.

Problema 140 Un juego consiste en lanzar un dado repetidas veces. El juego se detiene bien porque parece un 1, bien porque decidimos detenerlo en cualquier lanzamiento antes de que haya aparecido el n´ umero 1. El premio que recibimos es el n´ umero de puntos que muestra la cara del dado en el ultimo lanzamiento. ¿Cu´ al es la estrategia m´ as conveniente para obtener el mayor premio? Problema 141 Sea X una variable aleatoria no negativa con funci´ on de densidad f . Demostrar que Z ∞ E(X r ) = rxr−1 P (X > x)dx. 0

3.1. ESPERANZA DE UNA VARIABLE ALEATORIA

29

Problema 142 Sea X una variable aleatoria continua con media µ y funci´ on de distribuci´ on F . Demostrar que Z µ Z ∞ F (x)dx = (1 − F (x))dx. −∞

µ

Problema 143 Una caja contiene a bolas blancas y b bolas negras, con a > 0 y b > 0. Se extrae una bola al azar y si es negra el proceso termina. Si la bola es blanca se reintegra a la caja junto con otra bola blanca y el proceso continua. Calcular la probabilidad de que el proceso termine en la k-´esima extracci´ on. Calcular la probabilidad de que el n´ umero de extracciones sea finito. Obtener el n´ umero esperado de extracciones. Problema 144 (Un camino aleatorio en la recta) Un punto se desplaza sobre el eje x a derecha o izquierda de unidad en unidad. La probabilidad de que se desplace a la derecha es p y 1 − p de que lo haga a la izquierda. Si la variable X representa la posici´ on del punto despu´es de n desplazamientos, partiendo de 0, obtener su distribuci´ on de probabilidad y calcular E(X) y var(X). Problema 145 (N´ umero esperado de parejas restantes) Se trata de un problema propuesto y resuelto por Daniel Bernoulli en el siglo XVIII. Se propon´ıa averiguar el n´ umero esperado de parejas que permanecen completas, de un total de N parejas iniciales, despu´es de la muerte de m sus miembros. Problema 146 Sea X una variable aleatoria discreta con soporte DX = {x1 , x2 , . . . , xm }. Calcular p l´ım n E(X n ). n→∞

Problema 147 Se repite indefinida e independientemente un experimento cuyo resultado es E (´exito), con probabilidad p, o F (fracaso), con probabilidad q = 1 − p. Denotamos por ωn el resultado de la n-´esima repetici´ on. Sea T la variable que designa el n´ umero m´ınimo de repeticiones necesarias para alcanzar r ´exitos consecutivos. Demostrar que P (T = ∞) = 0 y calcular E(T ). Problema 148 Si g(x) es una funci´ on creciente no negativa de x, probar que P (X ≥ a) ≤

E(g(X)) . g(a)

Problema 149 Sea X una variable aleatoria con soporte DX = {0, 1, . . . , n} tal que E(X) = var(X) = 1. Demostrar que para cualquier natural k, P (X ≥ k + 1) ≤

1 . k2

x Problema 150 Tenemos 10 pares de zapatos y elegimos al azar 4 zapatos. ¿Cu´ al es la probabilidad de que no hayamos elegido ning´ un par? Si X es una variable aleatoria que representa el n´ umero de pares elegidos, obtener el n´ umero medio de pares entre los 4 zapatos elegidos. Problema 151 La funci´ on de densidad de la variable aleatoria X viene dada por  1   + cx3 , x ∈] − 1, 1[; 2 f (x) =   0, en el resto.

CAP´ITULO 3. ESPERANZA

30

1. Determinar los valores de c para que f (x) sea una densidad. 1

2. Calcular, si existen, los momentos de orden 1 y 2 de la variable Y = |X|− 2 . Problema 152 Definamos la funci´ on f (x) =



ax−(s+1) , si x > r; 0, si x ≤ r,

con r, s > 0. 1. Determinar a para que f (x) sea una funci´ on de densidad de probabilidad. 2. Si la variable aleatoria X tiene por densidad f (x), ¿para qu`e valores de s existir´ a su esperanza? Problema 153 Un autob´ us tiene en su recorrido 15 paradas. Supongamos que en la primera parada suben 20 personas. Cada una de ellas elige al azar e independientemente de las otras en cu´ al de las 14 paradas restantes quiere bajar. 1. Si Xi es una variable aleatoria que vale 1 si alguna de las personas baja en la parada i y 0 en caso contrario, calcular su distribuci´ on de probabilidad. 2. Calcular el n´ umero medio de paradas que debe realizar el autob´ us para que bajen todos los pasajeros. Problema 154 Elegimos un punto uniformemente en el c´ırculo centrado en cero y de radio uno. Supongamos que denotamos por Θ la variable aleatoria que nos da el a ´ngulo aleatorio asociado a este punto y por X la variable aleatoria que nos da la abscisa del punto. Se pide: 1. ¿Cu´ al es la distribuci´ on de Θ? Hay que obtener tanto la funci´ on de densidad como la funci´ on de distribuci´ on. 2. Determinar P(Θ ∈ [a, b]) para cualesquiera a y b con 0 ≤ a ≤ b ≤ 2π. 3. Determinar la funci´ on de distribuci´ on de la variable X. 4. Determinar la funci´ on de densidad de la variable X. 5. Calcular E(X). 6. Calcular E(|X|). Problema 155 Sea X una variable aleatoria con funci´ on de densidad  0, si x ≤ 0, f (x) = e−x xn , si x > 0, n! donde n es un natural. Demostrar que se verifica la siguiente desigualdad P (0 < X < 2(n + 1)) >

n . n+1

Problema 156 En una fila de 15 butacas de un cine se sientan aleatoriamente 7 mujeres y 8 hombres. Por t´ermino medio, ¿cuantas parejas de asientos adyacentes est´ an ocupadas por personas de distinto sexo.

3.1. ESPERANZA DE UNA VARIABLE ALEATORIA

31

Problema 157 Un reba˜ no de ovejas es sometido a examen para detectar aquellas que padecen determinada enfermedad. Sabemos que cada una de ellas la padece con probabilidad p e independientemente de las otras. La detecci´ on de la enfermedad requiere un an´ alisis de sangre y si se mezcla la sangre de n ovejas, basta que una de ellas padezca la enfermedad para que el an´ alisis d´e positivo. Como el reba˜ no es grande se plantean dos posibles estrategias: 1. examinar los animales uno a uno y llevar a cabo tantos an´ alisis como animales tiene el reba˜ no, o 2. examinar a los animales por grupos de n elementos, si el an´ alisis es negativo todos los del grupo est´ an sanos, pero si es positivo habr´ a que analizar uno a uno todos los animales del grupo. Determinar cu´ al de las dos estrategias es mejor porque conduce a un n´ umero menor de an´ alisis. Problema 158 Una variable aleatoria toma valores enteros positivos con probabilidades decreciendo en progresi´ on geom´etrica. Elegir el primer t´ermino y la raz´ on de la progresi´ on para que E(X) = 10, y calcular, bajo dichas condiciones, P (X ≤ 10). Problema 159 Para simular una moneda correcta a partir de una sesgada en la que la probabilidad de obtener una cara vale α 6= 1/2 podemos proceder como sigue. Lanzamos dos veces la moneda e interpretamos C+ como cara y +C como cruz. Si no aparece ninguno de estos dos resultados, repetimos los pares de lanzamientos hasta que aparezca alguno de ellos. Encontrar la distribuci´ on de probabilidad y la esperanza del n´ umero de pares de lanzamientos necesarios para poder tomar una decisi´ on (cara o cruz) y comprobar que, en efecto, se trata de la simulaci´ on de una moneda correcta. Problema 160 Determinar el valor esperado de la variable X definida en el problema 110. Problema 161 Sea X una variable aleatoria no negativa. Demostrar que [E(X)]1/2 ≥ E(X 1/2 ). Problema 162 Sea X el n´ umero de pruebas de Bernoulli necesarias para obtener un ´exito y un fracaso. Determinar la distribuci´ on de probabilidad de X y calcular E(X) y var(X). Problema 163 Determinar la media y la varianza de la variable X definida en el problema 83. Problema 164 (Examen 3-2-2004) Un jugador puede apostar a cualquiera de los n´ umeros enteros entre 1 y 6. Entonces lanza 3 dados y si aparece el n´ umero que eligi´ o, recibe como premio su apuesta multiplicada por el n´ umero de dados que lo muestran y adem´ as le devuelven lo que apost´ o. En otro caso pierde su dinero. ¿Cu´ al es la ganancia esperada de este juego? Problema 165 (Examen 8-6-2004) La variable aleatoria X, que toma valores en el intervalo [0,2], tiene por densidad la recta que pasa por (2,0) con pendiente negativa. Obtener su funci´ on de densidad f (x) y calcular P (|X − E(X)| ≤ 1/2). ¿Qu´e cota obtendr´ıamos para esta probabilidad si utiliz´ aramos la desigualdad de Chebychev? Problema 166 (Examen 8-6-2004) Tenemos una urna con 12 bolas numeradas de 1 a 12. Extraemos dos bolas y denotamos por X1 y X2 los valores que observamos en la primera y en la segunda extracci´ on. Sea X la variable definida como el m´ aximo de las dos extracciones. Se pide: 1. La funci´ on de distribuci´ on de la variable X si suponemos que las dos extracciones se realizan con reemplazamiento.

CAP´ITULO 3. ESPERANZA

32

2. La funci´ on de distribuci´ on de la variable X si suponemos que no hay reemplazamiento entre las extracciones sucesivas. 3. La media de la variable X en las dos situaciones anteriores. Problema 167 (Examen 1-9-2004) A partir de las variables aleatorias U1 , . . . , Un , independientes y todas ellas con distribuci´ on uniforme en el intervalo [0,1], se obtienen las variables Yi = −logUi , para i = 1, · · · , n. a) ¿Qu´e distribuci´ on siguen las variables Y1 , · · · , Yn ? ¿Son independientes? P b) Sea Y¯ = n1 ni=1 Yi . Calcular E(Y ), V ar(Y ) y demostrar que, ∀k > 0,

l´ım P (|Y − 1| > k) = 0.

n→∞

c) ¿Qu´e distribuci´ on de probabilidad sigue Y ? Problema 168 Determinar la media y varianza de la variable X definida en el problema 75.

3.2.

Esperanza de un vector aleatorio

Problema 169 Las variables aleatorias X e Y tienen media 0, varianza 1 y su coeficiente de correlaci´ on es ρ, Encontrar la media y la varianza de Z = X − ρY y sus coeficientes de correlaci´ on con X e Y . Problema 170 Las variables aleatorias X1 , X2 , . . . , Xm+n , n > m, son independientes e id´enticamente con varianza com´ un, σ 2 , finita. Calcular el coeficiente de correlaci´ on de Pn distribuidas P m+n U = i=1 Xi y V = j=m+1 Xj .  Problema 171 Una urna contiene 2n bolas numeradas de 1 a n, de forma que hay ni bolas con el n´ umero i. Si extraemos m sin reemplazamiento y por S denotamos la suma de sus n´ umeros, calcular E(S) y var(S).

Problema 172 (Un camino aleatorio en el plano) Un punto se desplaza sobre el plano con movimientos independientes de manera que en cada movimiento la distancia que recorre es siempre la misma, por ejemplo una unidad, pero la direcci´ on es aleatoria determinada por un a ´ngulo cuya orientaci´ on respecto la posici´ on del punto se elige al azar en el intervalo [0, 2π]. Si D es la distancia del punto a su posici´ on original despu´es de n movimientos, calcular E(D2 ). Problema 173 (N´ umero esperado de coincidencias) Recordemos que el llamado problema de las coincidencias consiste, en una de sus m´ ultiples versiones, en n individuos que al abandonar una fiesta recogen sus sombreros al azar. Si por X denotamos el n´ umero de individuos que han cogido su propio sombrero (coincidencias), calcular E(X) y var(X). Problema 174 Se realizan n pruebas independientes de un P experimento que tiene k resultados posibles A1 , A2 , . . . , Ak con probabilidades p1 , p2 , . . . , pk , pi = 1. Sea Y la variable n´ umero de resultados que no han aparecido en ninguna de las n pruebas. 1. ¿Qu´e distribuci´ on tiene cada una de las variables Xj que indica el n´ umero de veces que ha sucedido Aj ? 2. ¿Cu´ al es el valor esperado de Y ?

3.2. ESPERANZA DE UN VECTOR ALEATORIO

33

3. Comprobar que los valores de p1 , . . . , pk que hacen m´ınima E(Y ) son p1 = p2 = · · · = pk = 1/k. Problema 175 Sean X e Y dos variables aleatorias id´enticamente distribuidas, obtener cov(X+ Y, X − Y ). Problema 176 Sean X1 , X2 , . . . una familia de variables aleatorias independientes con la misma media µ y la misma varianza σ2 , y sea Yn = Xn + Xn+1 + Xn+2 . Hallar, para j ≥ 0, cov(Yn , Yn+j ). Problema 177 Sea X1 , X2 , . . . una sucesi´ on de variables aleatorias continuas independientes e id´enticamente distribuidas. Sea N ≥ 2 el ´ındice correspondiente a la variable donde la sucesi´ on deja de ser decreciente, es decir, X1 ≥ X2 ≥ . . . ≥ XN −1 < XN . Demostrar que E(N ) = e. Sugerencia.- Obtener primero P (N ≥ n). Problema 178 Sean X e Y el n´ umero de ´exitos y fracasos, respectivamente, cuando llevamos a cabo n pruebas Bernoulli con probabilidad de ´exito p. Calcular el coeficiente de correlaci´ on entre ambas, ρXY . Problema 179 Lanzamos tres veces consecutivas una moneda y definimos las variables aleatorias X ={n´ umero de caras en los dos primeros lanzamientos} e Y ={n´ umero de caras en los dos u ´ ltimos lanzamientos}. Obtener la distribuci´ on de probabilidad conjunta de X e Y , sus marginales y el coeficiente de correlaci´ on entre ambas. Problema 180 Obtener el coeficiente de correlaci´ on del vector (X, Y ) definido en el problema 94. Problema 181 ( ) Sean X e Y dos variables aleatorias indicatrices independientes tales que P (X = 1) = p y P (Y = 1) = q. Halla E[(X − Y )2 ] en t´erminos de p y q. Problema 182 ( ) La densidad conjunta de las variables X e Y es ( c(y 2 − x2 )e−y si −y < x < y , y > 0, fXY (x, y) = 0 en el resto, donde c es una constante. 1. Demuestra que Y sigue una distribuci´ on gamma y deduce que c = 81 . 2. Halla la densidad de 4Y 3 . 3. Obt´en las esperanzas y varianzas de X e Y . Problema 183 (Un ejemplo de fabricaci´ on simple) Un fabricante de bolas de golf est´ a evaluando un nuevo sistema de producci´ on. En este nuevo proceso de fabricaci´ on la probabilidad de una bola defectuosa, que no puede ser vendida, es de 0.05 mientras que en el procedimiento que actualmente utiliza es de 0.08. Con el procedimiento que utiliza el coste de producci´ on de cada unidad es de 40 pesetas por bola mientras que en el nuevo el coste de cada bola es de 60 pesetas. Las bolas se venden a 125 pesetas cada una. Si el fabricante desea tener una ganancia esperada m´ axima, ¿qu´e procedimiento de fabricaci´ on ha de utilizar? Problema 184 (Variables discretas sim´ etricas) Supongamos X una variable aleatoria discreta tal que P(X = a − x) = P(X = a + x) para cualquier x. Determinar la esperanza de la variable X.

CAP´ITULO 3. ESPERANZA

34

Problema 185 (Examen 11-2-2000) Sean √ U1 y U2 dos variables aletorias independientes ambas uniformes en [0,1]. Definimos X = U1 e Y = 2XU2 . 1. Obtener la densidad conjunta de (X, Y ) 2. Obtener las densidades marginales de X e Y y sus esperanzas. 3. Si W es una variable Bernoulli tal que P (W = 1) = P (0 ≤ Y ≤

√ X), calcular E(W ).

Problema 186 ( ) El peso de un tumor tras un tiempo t, Wt , viene dado por la f´ ormula Wt = XetY donde X e Y son variables aleatorias independientes, X sigue una distribuci´ on gamma con media 2 y varianza 1 e Y est´ a uniformemente distribuida en el intervalo (1, 1,5). Halla E(Wt ) y var(Wt ). Problema 187 (Krief y Levy, p´ agina 221) Sean X, Y y Z tres variables aleatorias. Se supone que las tres parejas (X, Y ), (Y, Z) y (Z, X) tienen el mismo coeficiente de correlaci´ on r. Probar que se verifica 1 (3.1) r≥− . 2 Se podr´ a, en un primer momento, suponer que las variables aleatorias X, Y y Z tienen la misma varianza. Problema 188 (Krief y Levy, p´ agina 223) Sea X una variable aleatoria continua que admite una funci´ on de densidad f (x) y que tiene momentos finitos de los dos primeros o ´rdenes. Se supone que la funci´ on f es par, es decir, que se tiene que para todo x, f (x) = f (−x).

(3.2)

Probar que: 1. Probar que E(X) = 0. 2. Probar que el coeficiente de correlaci´ on r entre X y |X| es nulo. ¿Conclusi´ on? Problema 189 ( ) Un terremoto de magnitud M libera una cantidad de energ´ıa X tal que M = ln X. Para terremotos de magnitud mayor que 3 sup´ on que M − 3 tiene una distribuci´ on exponencial de media 2. 1. Halla E(M ) y V ar(M ) para un terremoto de magnitud mayor que 3 2. Para un terremoto como el del apartado anterior halla la densidad de X 3. Consideremos dos terremotos, que han sucedido independientemente, ambos de magnitud mayor que 3. ¿Cu´ al es la probabilidad de que la magnitud del terremoto m´ as peque˜ no sea mayor de 4? Problema 190 Un accidente tiene lugar en un punto X ∼ U (0, L). En el instante del accidente una ambulancia est´ a en una localizaci´ on Y tambi´en distribuida uniformemente en la carretera de longitud L. Halla la distancia esperada E(| X − Y |) entre el accidente y la ambulancia. Problema 191 (Uno de ascensores) Un edificio tiene 10 pisos. 12 personas suben a un ascensor y cada una de ellas elige aleatoriamente el piso en donde quiere bajar (es decir, elige con la misma probabilidad cada uno de los posibles pisos). Adem´ as hace su elecci´ on independientemente de los dem´ as. ¿Cu´ al es el n´ umero medio de pisos en los que ha de parar el ascensor con objeto de que baje una o m´ as personas?

35

3.3. ESPERANZA CONDICIONADA

3.3.

Esperanza condicionada

Problema 192 Una m´ aquina fabrica fibras de longitud aleatoria X. Para medir la desigualdad entre las longitudes de las fibras fabricadas se utiliza el coeficiente λ=

a′′ − a′ , a

donde a = E(X), a′′ = E(X | X > a) y a′ = E(X | X < a). Si X ∼ N (a, σ 2 ), encontrar la relaci´ on entre λ, a y σ. Problema 193 (Un problema de pastillas. La distribuci´ on Hipergeom´ etrica negativa) Una caja contiene pastillas de dos tipos: grandes y peque˜ nas. Cada pastilla grande equivale a dos peque˜ nas. Cada d´ıa el paciente debe tomar una de las pastillas, que elige al azar. Si es de las grandes la parte en dos, toma una mitad y devuelve la otra a la caja. Si la pastilla es peque˜ na la toma sin m´ as. A todos los efectos las mitades devueltas a la caja son consideradas como pastillas peque˜ nas. ¿Cu´ al es el n´ umero esperado de pastillas peque˜ nas que quedar´ an en la caja cuando las grandes se hayan terminado? Problema 194 El n´ umero de clientes en la cola de la caja de un supermercado sigue una distribuci´ on de Poisson con par´ ametro λ. El tiempo que tarda cada cliente en ser atendido sigue una Exponencial de par´ ametro λ. Calcular el tiempo medio que debemos esperar en la cola. Problema 195 El n´ umero de pasajeros que espera el tren en cierta estaci´ on en un instante t es una variable aleatoria Poisson de par´ ametro λt. Si el tiempo de llegada del tren se distribuye uniformemente en el intervalo [0, T ], ¿cu´ al es el n´ umero medio de pasajeros que subir´ a al tren? Obtener tambi´en su varianza. Problema 196 Un minero atrapado en una mina tiene tres posibles caminos para tratar de escapar de la mina. El primero le conduce al exterior despu´es de 3 horas. El segundo le conduce de nuevo al interior de la mina despu´es de un recorrido de 5 horas. El tercero le conduce tambi´en al interior de la mina, pero despu´es de 7 horas de recorrido. Si el minero echa a suertes (igual probabilidad) el camino por el que tratar de escapar, ¿cu´ al ser´ a el tiempo de medio que tardar´ a en conseguirlo? Problema 197 Sea {Xj }j≥1 una sucesi´ on de variables aleatorias uniformes en el intervalo [0, 1]. Calcular E(N ) siendo   n   X Xj > 1 . N = m´ın n;   j=1

Problema 198 El vector aleatorio (X, Y ) tiene por densidad conjunta  1   ye−yx, 1 < y < 3, x > 0; 2 fXY (x, y) =   0, en el resto.

Hallar E(X) y var(X) haciendo uso de E(X|Y ) y de var(X|Y ).

36

CAP´ITULO 3. ESPERANZA

Problema 199 El vector aleatorio (X, Y ) tiene por densidad conjunta  2e−2x   , 0 ≤ x < +∞, 0 ≤ y ≤ x; x fXY (x, y) =   0, en el resto.

Calcular cov(X, Y ). Sugerencia.- La obtenci´ on de E(Y ) puede resultar m´ as sencilla a trav´es de E(Y ) = E[E(Y |X)]. Problema 200 Tenemos un lote de N pastillas y antes de comercializarlo lo sometemos al siguiente control de calidad. Fijamos un umbral c (c < n) y a continuaci´ on tomamos una muestra de n pastillas del total de N que componen el lote. Sea X la variable aleatoria que nos da el n´ umero de pastillas en mal estado en la muestra, si X ≤ c entonces sustituimos las X pastillas defectuosas por pastillas en buen estado y comercializamos, sin m´ as inspecci´ on, el lote. En el caso en que X > c entonces se muestrean todas y cada una de las pastillas que componen el lote y se sustituyen por pastillas en buen estado, comercializ´ andolo a continuaci´ on. En este caso el lote no tiene ninguna pastilla en mal estado. Si la probabilidad de que el proceso de fabricaci´ on produzca una pastilla en mal estado es p, ¿qu´e n´ umero medio de pastillas en mal estado estamos lanzando al mercado? Problema 201 Elegimos al azar dos puntos en el intervalo [0, a]. Queremos conocer, a) Area media del rect´ angulo que tiene por lados las correspondientes longitudes. b) Area media de dicho rect´ angulo si ambos puntos son mayores que a/2. Problema 202 Un trabajador est´ a encargado del correcto funcionamiento de n m´ aquinas situadas en linea recta y distantes una de otra l metros. El trabajador debe repararlas cuando se aver´ıan, cosa que sucede con igual probabilidad para todas ellas e independientemente de una a otra. El operario puede seguir dos estrategias: 1. acudir a reparar la m´ aquina estropeada y permanecer en ella hasta que otra m´ aquina se aver´ıa, desplaz´ andose entonces hacia ella, o 2. situarse en el punto medio de la linea de m´ aquinas y desde all´ı acudir a la averiada, regresando nuevamente a dicho punto cuando la aver´ıa est´ a resuelta. Si X es la distancia que recorre el trabajador entre dos aver´ıas consecutivas, ¿c´ ual de ambas estrategias le conviene m´ as para andar menos? Problema 203 Sean X e Y dos variables aleatorias. Si E(X|Y ) = 10−Y y E(Y |X) = 7−X/4, obtener su coeficiente de correlaci´ on. Problema 204 Se pide determinar la media y la varianza de la variable X definida en el problema 111. Problema 205 Calcular el coeficiente de correlaci´ on entre el mayor y el menor valor obtenidos al lanzar dos dados. Problema 206 Obtener el coeficiente de correlaci´ on entre las coordenadas de un punto elegido al azar en el c´ırculo unidad.

3.3. ESPERANZA CONDICIONADA

37

Problema 207 (Examen 8-6-2004) La variable aleatoria X se distribuye exponencialmente con par´ ametro Y , que es a su vez una variable aleatoria uniforme en [1, 4]. Obtener la distribuci´ on conjunta de X e Y y la esperanza y la varianza de X. Problema 208 (Examen 1-9-2004) Sea X una variable aleatoria con distribuci´ on normal est´ andar (con media 0 y varianza 1) y sea I otra variable aleatoria, independiente de X y tal que P (I = 1) = P (I = 0) = 1/2. Se define la variable aleatoria Y mediante  X, si I = 1; Y = −X, si I = 0. 1. Calcular las probabilidades P (X < 1, Y > 1), P (X < 1) y P (Y > 1). ¿Son independientes X e Y ? Justifica la respuesta. 2. Demostrar que Y sigue una distribuci´ on normal est´ andar. 3. Calcular E[XY |I = 1] y E[XY |I = 0] y demostrar que Cov(X, Y ) = 0. Problema 209 (Examen 21-6-2005) Sobre un c´ırculo cuyo radio R es aleatorio con funci´ on de densidad  2 r   , r ∈ [0, 3]; 9 fR (r) =   0 en el resto, elegimos un punto al azar. Si X designa la distancia del punto al origen, obtener 1. La funci´ on de distribuci´ on y la funci´ on de densidad de X|R = r. 2. La media y la varianza de X.

38

CAP´ITULO 3. ESPERANZA

Cap´ıtulo 4

Convergencia de sucesiones de variables aleatorias 4.1.

Tipos de convergencia

Problema 210 Consideremos la variable aleatoria X ∼ U (0, 1) y definamos la sucesi´ on Xn = L X/n, n ≥ 1. Comprobar que Xn → Y , siendo Y una variable aleatoria degenerada, P (Y = 0) = 1. Problema 211 La sucesi´ on de variables aleatorias {Xn }n≥1 tales que P (Xn = 1 − 1/n) = P (Xn = 1 + 1/n) = 1/2, convergen en ley a la variable aleatoria X tal que P (X = 1) = 1. ¿Tienden sus funciones de probabilidad a una funci´ on de probabilidad? Problema 212 Sean Xj , j = 1, . . . , n, variables aleatorias independientes, todas ellas U (0, 1). Definimos Yn = m´ın(X1 , . . . , Xn ),

Zn = m´ax(X1 , . . . , Xn ),

Un = nYn ,

Vn = n(1 − Zn ).

Demostrar que cuando n → +∞, P

Yn −→ 0;

L

P

Un −→ U ;

Zn −→ 1;

P

Vn −→ V,

siendo U y V variables aleatorias exponenciales con par´ ametro λ = 1. Problema 213 Aplicar el anterior resultado a las variables aleatorias Xj , j = 1, . . . , n, independientes dos a dos y tales que P (Xj = −aj ) = P (Xj = aj ) =

1 . 2

¿Para qu´e valores de a es aplicable el resultado? Problema 214 Consideremos on deciPn x ∈ [0, 1] y sea ξn (x)={el n-´esimo d´ıgito de su expresi´ mal}. Definamos Sn (x) = k=1 ξk (x) y   2Sn (x) − 9n √ An (y) = x; 0, y definamos Y = cT . Para k ∈ N , las variables aleatorias Xk se definen mediante,   −1, si −1 < Y < −1/k; 0, si −1/k ≤ Y < 1/k; Xk =  1, si 1/k ≤ Y < 1.

Demostrar que Xk converge en ley a la variable aleatoria X con funci´ on de probabilidad fX (−1) = fX (1) = 1/2 y fX (x) = 0, x ∈ / {−1, 1}.

Problema 217 (Examen 1-9-2004) Una empresa que alquila coches con conductor ha observado que el n´ umero de kil´ ometros por d´ıa de alquiler que se hacen con un determinado tipo de veh´ıculo sigue una distribuci´ on N (200, 5625). a) ¿Cu´ al es la probabilidad de que en 30 d´ıas se hagan m´ as de 5.000 kil´ ometros? ¿Qu´e hip´ otesis hay que asumir? b) La compa˜ n´ıa revisa el estado del veh´ıculo si en un mismo d´ıa hace m´ as de 350 km. Calcular el n´ umero esperado de d´ıas que la compa˜ n´ıa puede alquilar el veh´ıculo antes de tener que revisarlo.

4.2.

Leyes de los Grandes N´ umeros

Problema 218 Demostrar que la independencia de las variables en la ley d´ebil de los grandes n´ umeros puede relajarse exigiendo solamente independencia dos a dos y acotaci´ on de las varianzas. Es decir, si Xj , j = 1, . . . , n, verifican que E(Xj ) = µj y var(Xj ) = σj2 son finitas, entonces P X n − µn −→ 0, donde

n

µn =

1X µj , n j=1

n

Xn =

las Xj son independientes dos a dos y σj2 ≤ M , ∀j.

1X Xj n j=1

Problema 219 Sea Sn el n´ umero de ´exitos en n pruebas Bernoulli con probabilidad de ´exito p en cada prueba. Encontrar una cota para P (|Sn /n − p| ≥ ǫ) que no dependa de p. Problema 220 Tenemos dos monedas, una correcta y otra con probabilidad de cara p = 3/4. Elegimos al azar una de las dos y realizamos una serie de lanzamientos. Despu´es de observar el resultado de un gran n´ umero de ellos, ¿podemos saber la moneda elegida? ¿Cu´ al es el m´ınimo n´ umero de lanzamientos para poder saberlo con una probabilidad de al menos 95 %?

´ CARACTER´ISTICA 4.3. FUNCION

4.3.

41

Funci´ on caracter´ıstica

Problema 221 Probar que una variable aleatoria X es sim´etrica s´ı y s´ olo s´ı su funci´ on caracter´ıstica es real. Problema 222 Encontrar la distribuci´ on de las variables aleatorias que tiene por funci´ on caracter´ıstica X X 1) φ1 (t) = ak cos kt, 2) φ2 (t) = ak eiλk t . k≥0

4.4.

k≥0

Teorema Central de L´ımite

Problema 223 Ante la caja de un banco hay 60 personas que esperan recibir su salario, del que sabemos que es una cantidad aleatoria de media µ = 100 euros y desviaci´ on t´ıpica σ = 30 euros. Si los salarios son independientes de un asalariado a otro, queremos saber: 1. ¿Cu´ anto dinero debe tener la caja para, con probabilidad 0,95, poder pagar todos los salarios? 2. si la caja cuenta incialmente con 7,000 euros, ¿cu´ al es la probabilidad de que al final de los pagos queden en caja al menos 500 euros? Problema 224 Una empresa produce 10000 bolas de acero para rodamientos, siendo p = 0,05 la probabilidad de que una bola sea defectuosa. El proceso de producci´ on garantiza que las bolas son fabricadas independientemente unas de otras. Las bolas defectuosas son arrojadas a un recipiente cuya capacidad queremos determinar para que, con probabilidad 0.99, quepan en ´el todas las bolas defectuosas del proceso. Problema 225 Las bombillas utilizadas por cierto aparato pueden ser de dos clases, A y B. Las de la clase A tienen una duraci´ on media µA = 2000 horas con devsiaci´ on t´ıpica σA = 400 horas, mientras que las de la clase B tienen una duraci´ on media µB = 1800 horas con desviaci´ on t´ıpica σB = 500 horas. Se compran 200 bombillas de la clase A y 150 de la clase B. Calcular la probabilidad de que la duraci´ on media de la muestra de la clase A no supere en m´ as de 100 horas la duraci´ on media de la muestra de la clase B. Problema 226 Un colegio est´ a preparando la fiesta de graduaci´ on de sus 500 estudiantes. Se sabe, por lo ocurrido en otras ocasiones, que el 50 % de los estudiantes vienen acompa˜ nados de sus padres, el 30 % s´ olo por uno de ellos y el 20 % restante vienen solos. ¿Cuantos asientos hay que disponer para los padres si queremos que, con una probabilidad superior a 0.95, todos ellos puedan sentarse? Problema 227 Disponemos de un dado cargado en el que la probabilidad de obtener cualquiera de las caras es proporcional a su n´ umero de puntos. Jugamos con ´el pagando 4 euros por jugada y recibiendo como premio tantos euros como puntos tiene la cara obtenida al lanzarlo. Obtener la probabilidad aproximada de ir ganando al cabo de 100 jugadas. Problema 228 Probar con argumentos probabil´ısticos que e−n

n X nk

k=0

k!

n

−→

1 . 2

42

CAP´ITULO 4. CONVERGENCIA DE SUCESIONES DE VARIABLES ALEATORIAS

Problema 229 La fabricaci´ on de zumo de naranja se lleva a cabo por lotes de n envases. La caducidad en d´ıas de cada envase es una variable aleatoria de media µ = 20 y varianza σ 2 = 9. Obtener la media y la varianza de la caducidad media del lote y calcular el tama˜ no m´ınimo del lote para que con probabilidad 0,99 dicha caducidad media supere los 19 d´ıas. Problema 230 (Todav´ıa existen las pesetas) Un programa de contabilidad redondea las cantidades (expresadas en pesetas con una precisi´ on de dos decimales) a la peseta m´ as pr´ oxima de manera que las cantidades con c´entimos entre 0 y 49 quitamos los centimos correspondientes mientras que las cantidades con c´entimos entre 50 y 99 lo redondeamos a la unidad superior. Determinar la probabilidad aproximada que que el error acumulado en 100 transacciones sea mayor que 5 pesetas (por exceso o por defecto) suponiendo que el n´ umero de centimos que nos aparece en una cantidad cualquiera se distribuye uniformemente en {0, . . . , 99}. Problema 231 Dos compa˜ n´ıas a´ereas A y B ofrecen el mismo servicio en dos vuelos diferentes que salen al mismo tiempo (en definitiva, es igualmente probable que un pasajero coja uno de los vuelos). Supongamos que ambas compa˜ n´ıas compiten por un grupo de 400 pasajeros potenciales. La compa˜ n´ıa A vende un billete a cualquiera que se lo solicita y la capacidad de su avi´ on es de 230 pasajeros. Determinar la probabilidad aproximada de que A tenga un overbooking (que alg´ un pasajero tenga la desagradable sorpresa de tener un billete pero no tenga asiento). Problema 232 (Examen 8-6-2004) Se redondean 20 n´ umeros al entero m´ as cercano y despu´es se suman. Supongamos que los errores de redondeo son independientes y uniformemente distribuidos en el intervalo [− 12 , 21 ]. Se pide determinar la probabilidad de que la suma obtenida difiera de la suma de los 20 n´ umeros originales en m´ as de 3 unidades.

4.5.

Funci´ on generatriz de momentos

Problema 233

1. Halla la funci´ on generatriz de momentos de X ∼ P o(λ).

2. Obt´en la esperanza y la varianza de X. 3. Demuestra que si Xi ∼ P o(λi ) para i = 1, . . . , k y son variables independientes entonces Pk Pk S = i=1 Xi ∼ P o(λ), siendo λ = i=1 λi .

Problema 234 1. Demuestra que si Xi ∼ Exp(λ) para i = 1, . . . , k y son variables aleatoPk rias independientes entonces S = i=1 Xi ∼ Ga(k, λ). 2. Halla E(S) y V ar(S).

Problema 235 Sea X1 , . P . . Xn una muestra aleatoria de una N (µ, σ) demostrar que Xn ∼ n N (µ, √σn ), siendo Xn = n1 i=1 Xi .

Problema 236 Si Xi ∼ χ2ni para i = 1, . . . , k y son variables aleatorias independientes entonP P ces S = ki=1 Xi ∼ χ2n , siendo n = ki=1 ni .

Problema 237 Halla la funci´ on generatriz de momentos de X ∼ BN (r, p) y calcular E(X).

Cap´ıtulo 5

Ex´ amenes previos 5.1. 5.1.1.

1 de septiembre de 2004 Castellano

Problema 238 Supongamos que una caja contiene 5 monedas, cada una de ellas con distinta probabilidad de obtener una cara la lanzarla. Denotamos por pi dicha probabilidad para la moneda i, i = 1, 2, . . . , 5 y supongamos que p1 = 0, p2 = 1/4, p3 = 1/2, p4 = 3/4 y p5 = 1. a) Se selecciona al azar una moneda. Si la primera cara se obtiene al cuarto lanzamiento, ¿cu´ al es la probabilidad de que la i−´esima moneda haya sido la seleccionada? b) Si se lanza de nuevo la misma moneda, ¿cu´ al es la probabilidad de obtener una nueva cara? c) Si se hubiera obtenido una cruz en el primer lanzamiento, ¿cu´ al es la probabilidad de obtener una cara en el segundo lanzamiento? Problema 239 Sea X una variable aleatoria con distribuci´ on normal est´ andar (con media 0 y varianza 1) y sea I otra variable aleatoria, independiente de X y tal que P (I = 1) = P (I = 0) = 1/2. Se define la variable aleatoria Y mediante  X, si I = 1; Y = −X, si I = 0. 1. Calcular las probabilidades P (X < 1, Y > 1), P (X < 1) y P (Y > 1). ¿Son independientes X e Y ? Justifica la respuesta. 2. Demostrar que Y sigue una distribuci´ on normal est´ andar. 3. Calcular E[XY |I = 1] y E[XY |I = 0] y demostrar que Cov(X, Y ) = 0. Problema 240 Una empresa que alquila coches con conductor ha observado que el n´ umero de kil´ ometros por d´ıa de alquiler que se hacen con un determinado tipo de veh´ıculo sigue una distribuci´ on N (200, σ = 75). a) ¿Cu´ al es la probabilidad de que en 30 d´ıas se hagan m´ as de 5.000 kil´ ometros? ¿Qu´e hip´ otesis hay que asumir?

43

´ CAP´ITULO 5. EXAMENES PREVIOS

44

b) La compa˜ n´ıa revisa el estado del veh´ıculo si en un mismo d´ıa hace m´ as de 350 km. Calcular el n´ umero esperado de d´ıas que la compa˜ n´ıa puede alquilar el veh´ıculo antes de tener que revisarlo. Problema 241 A partir de las variables aleatorias U1 , . . . , Un , independientes y todas ellas con distribuci´ on uniforme en el intervalo [0,1], se obtienen las variables Yi = −logUi , para i = 1, · · · , n. a) ¿Qu´e distribuci´ on siguen las variables Y1 , · · · , Yn ? ¿Son independientes? P b) Sea Y¯ = n1 ni=1 Yi . Calcular E(Y ), V ar(Y ) y demostrar que, ∀k > 0,

l´ım P (|Y − 1| > k) = 0.

n→∞

c) ¿Qu´e distribuci´ on de probabilidad sigue Y ?

5.1.2.

Valenciano

Problema 242 Una caixa cont´e 5 monedes, cadascuna d’elles amb distinta probabilitat de mostrar una cara en ser llan¸cada. Si per pi denotem la probabilitat de cara per a la moneda i, tenim p1 = 0, p2 = 1/4, p3 = 1/2, p4 = 3/4 y p5 = 1. a) triem una moneda a l’atzar. Si la primera cara apareix en el quart llan¸cament, quina ´es la probabilitat de que la i−`esima moneda haja estat la seleccionada? b) Si llancem una altra vegada la mateixa moneda, quina ´es la probabilitat d’obtenir una cara? c) Si al primer llan¸cament hagu´erem tret una creu, quina hagu´es estat la probabilitat d’obtenir una cara en el segon llan¸cament? Problema 243 Siga X una variable aleat` oria amb distribuci´ o normal est´ andar (mitjana 0 i varian¸ca 1) i siga I una altra variable aleat` oria, independent d’X i tal que P (I = 1) = P (I = 0) = 1/2. Definim Y mitjan¸cant  X, si I = 1; Y = −X, si I = 0. 1. Calcular les probabilitats P (X < 1, Y > 1), P (X < 1) i P (Y > 1). S´ on independents X i Y ? Justifica la resposta. 2. Demostrar que Y segueix una distribuci´ o normal est´ andar. 3. Calcular E[XY |I = 1] i E[XY |I = 0] i demostrar que Cov(X, Y ) = 0. Problema 244 Una empresa de lloguer de cotxes amb ch´ ofer ha observat que el n´ umero de quil` ometres per dia de lloguer que fa un cert tipu de vehicle segueix una distribuci´ o N (200, σ = 75). a) Quina ´es la probabilidad de que un d’aquests vehicles fa¸ca m´es de 5.000 quil` ometres en 30 dies? Quina hip` otesi hi ha que assumir? b) La companyia revisa l’estat del vehicle si en un mateix dia fa m´es de 350 qm. Calcular el nombre esperat de dies que la compa˜ n´ıa pot llogar el cotxe sense haver de revisar-lo .

45

5.2. 3 DE FEBRERO DE 2004

Problema 245 A partir de les variables aleat` ories U1 , . . . , Un , independents i totes elles amb distribuci´ o uniforme en l’interval [0,1], obtenim les variables Yi = −logUi , i = 1, · · · , n. a) Quina distribuci´ o segueixen les variables Y1 , · · · , Yn ? S´ on independents? P n b) Siga Y¯ = n1 i=1 Yi . Calcular E(Y ), V ar(Y ) i demostrar que, ∀k > 0,

l´ım P (|Y − 1| > k) = 0.

n→∞

c) Quina distribuci´ o de probabilitat segueix Y ?

5.2. 5.2.1.

3 de febrero de 2004 Castellano

Problema 246 Un jugador puede apostar a cualquiera de los n´ umeros enteros entre 1 y 6. Entonces lanza 3 dados y si aparece el n´ umero que eligi´ o, recibe como premio su apuesta multiplicada por el n´ umero de dados que lo muestran y adem´ as le devuelven lo que apost´ o. En otro caso pierde su dinero. ¿Cu´ al es la ganancia esperada de este juego? Problema 247 Un taxi se ve involucrado en un accidente nocturno. En la ciudad hay dos compa˜ n´ıas de taxis, los taxis Negros y los taxis Blancos. Se sabe que el 85 % de los taxis de la ciudad son Negros y el 15 % restante son Blancos. Un testigo del accidente afirma que el taxi involucrado era Blanco y la fiabilidad de su testimonio es del 80 %, es decir, es capaz de identificar correctamente el color del taxi el 80 % de las veces. 1. Calcula la probabilidad de que el taxi accidentado fuera el Blanco, dado que el testigo afirma que lo era, y comp´ arala con la respuesta del testigo. 2. Supongamos que el 100p % de los taxis son Blancos, con 0 ≤ p ≤ 1, y que la fiabilidad del testigo contin´ ua siendo del 80 %. Estudia la sensibilidad a los datos de la respuesta anterior viendo como var´ıa ´esta en funci´ on de p. ¿A partir de qu´e valor de p la anterior probabilidad supera 0.5? 3. El an´ alisis anterior puede completarse permitiendo que la fiabilidad del testigo sea variable, 100q %, con 0 ≤ q ≤ 1. Determina la relaci´ on que se debe dar entre p y q para que la probabilidad pedida supere 0.5. Problema 248 El holand´es Christian Huygens public´ o en 1657 uno de primeros libros sobre Probabilidad que se conocen, De Ratiociniis in Ludo Aleae (Del Razonamiento en los Juegos de Azar), en el que planteaba una serie de problemas. El que se conoce como segundo problema de Huygens lo enunciamos a continuaci´ on. Tres jugadores A, B y C participan en el siguiente juego. Una urna contiene a bolas blancas y b negras. Los jugadores, en el orden ABCABC . . ., extraen una bola con reemplazamiento hasta que uno de ellos obtiene una bola blanca y gana. Encontrar la probabilidad de ganar para cada jugador. Problema 249 Elegimos un punto uniformemente en el c´ırculo centrado en cero y de radio uno. Supongamos que denotamos por Θ la variable aleatoria que nos da el a ´ngulo aleatorio asociado a este punto y por X la variable aleatoria que nos da la abscisa del punto. Se pide:

´ CAP´ITULO 5. EXAMENES PREVIOS

46

1. ¿Cu´ al es la distribuci´ on de Θ? Hay que obtener tanto la funci´ on de densidad como la funci´ on de distribuci´ on. 2. Determinar P(Θ ∈ [a, b]) para cualesquiera a y b con 0 ≤ a ≤ b ≤ 2π. 3. Determinar la funci´ on de distribuci´ on de la variable X. 4. Determinar la funci´ on de densidad de la variable X. 5. Calcular E(X). 6. Calcular E(|X|).

5.2.2.

Valenciano

Problema 250 Un jugador pot apostar a qualsevol dels enters entre 1 y 6, tots dos inclosos. Triat el n´ umero llan¸ca 3 daus, i si algun d’ells mostra el n´ umero que ha triat rep com a premi tantes vegades el que va apostar com daus mostren el n´ umero, i a m´es a m´es li tornen l’aposta. ¿Quin ´es el guany esperat en aquest joc? Problema 251 Un taxi t´e un accident nocturn en una ciutat on hi ha dues companyes de taxis, els Negres i els Blancs. Sabem que el 85 % dels taxis s´ on Negres i la resta blancs. Un testimoni de l’accident, que ´es capa¸c d’identificar correctament el color del taxi en el 80 % de les ocasions, afirma que el taxi era Blanc. 1. Calcula la probabilitat de que el taxi accidentat fos blanc, donat que el testimoni aix´ı ho afirma, i compara-la amb el que diu el testimoni. 2. Suposem ara que el 100p % dels taxis s´ on Blancs, on 0 ≤ p ≤ 1, i que la fiabilitat del testimoni continua essent del 80 %. Estudia la sensibilitat de la probabilitat obtinguda a l’apartat anterior mitjan¸cant la seua variaci´ o en funci´ o de p. A partir de quin valor de p aquesta probabilitat supera 0,5? 3. L’anterior an` alisi pot completar-se permetent que la fiabilitat del testimoni siga variable, 100q %, on 0 ≤ q ≤ 1. Determina la relaci´ o que deuen tenir p i q per a que l’esmentada probabilitat supere 0,5. Problema 252 L’holand´es Christian Huygens va publicar en 1657 un dels primers llibres sobre Probabilitat, De Ratiociniis in Ludo Aleae (Del Raonament en els Jocs d’Atzar), on proposava un seguit de problemes. El conegut com a segon problema de Huygens l’enunciem tot seguit. Tres jugadors A, B i C participen en el seg¨ uent joc. Una urna t´e a bolas blanques i b negres. Els jugadors, en l’ordre ABCABC . . ., trauen una bola amb reempla¸cament fins que un d’ells trau una bola blanca i guanya. Trobar la probabilitat de guanyar per a cada jugador. Problema 253 Triem un punt a l’atzar dintre del cercle unitat centrat en zero. Designem mitjan¸cant Θ i X, respectivament, l’angle aleatori i l’abscissa associats al punt. Es demana: 1. La distribuci´ on de Θ (cal obtenir tant la funci´ o de densitat com la funci´ o de distribuci´ o). 2. Determinar P(Θ ∈ [a, b]), ∀a, b amb 0 ≤ a ≤ b ≤ 2π. 3. Determinar la funci´ o de distribuci´ o de la variable X.

47

5.3. 3 DE SEPTIEMBRE DE 2005

4. Determinar la funci´ o de densitat de la variable X. 5. Calcular E(X). 6. Calcular E(|X|).

5.3. 5.3.1.

3 de septiembre de 2005 Castellano

Problema 254 Tenemos 10 pares de zapatos y elegimos al azar 4 zapatos. ¿Cu´ al es la probabilidad de que no hayamos elegido ning´ un par? Si X es una variable aleatoria que representa el n´ umero de pares elegidos, obtener el n´ umero medio de pares entre los 4 zapatos elegidos. Ayuda.- Para el c´ alculo de E(X) puede ayudar definir variables Xj que valen 1 si el par j ha sido escogido y 0 en caso contrario. Problema 255 Cuando una corriente de I amperios pasa a trav´es de una resistencia de R ohmios, la potencia generada viene dada por W = I 2 R vatios. Supongamos que I y R son variables aleatorias independientes con densidades   6x(1 − x), si 0 ≤ x ≤ 1; fI (x) =  0, fuera. y

fR (y) = Hallar la densidad de W .

  2y, si 0 ≤ y ≤ 1; 

0,

fuera.

Problema 256 La variable aleatoria Y se distribuye Exp(1). Definimos   1, si Y < ln n; Xn =  0, en caso contario.

Obtener la funci´ on de distribuci´ on de Xn y estudiar la convergencia en ley de las Xn .

Problema 257 La probabilidad de que un virus inform´ atico haya infectado nuestro ordenador es 0, 1. Si el ordenador est´ a infectado, un sistema antivirus detecta la infecci´ on con probabilidad x = 0, 95, mientras que en caso de no infecci´ on el sistema detecta falsas infecciones con probabilidad y = 0, 03. Interesa que el sistema antivirus tenga un elevado valor predictivo={probabilidad de que el ordenador est´e infectado cuando el antivirus detecta una infecci´ on}. Calcularlo a partir de los datos anteriores. Si queremos aumentarlo, ¿donde hemos de dirigir nuestros esfuerzos, a aumentar x o a rebajar y?

5.3.2.

Valenciano

Problema 258 Tenim 10 parelles de sabates i triem a l’atzar 4 sabates. Quina ´es la probabilitat de no haver triat cap parella? Si X ´es la variable aleat` oria que representa el n´ umero de parelles que hem triat, obtenir la mitjana del n´ umero de parelles entre les 4 sabates que hem triat. Ajut.- Per al c` alcul d’E(X) pot ajudar definir variables Xj que valen 1 si el parell de sabates j ha estat triat i 0 en cas contrari.

´ CAP´ITULO 5. EXAMENES PREVIOS

48

Problema 259 Un corrent d’intensitat I ampers en creuar una resist`encia de R ohms, genera una pot`encia de W = I 2 R vats. Suposem que I i R s´ on variables aleat` ories independents amb densitats   6x(1 − x), si 0 ≤ x ≤ 1; fI (x) =  0, fora. i

fR (y) = Trobar la densitat de W .

  2y, si 0 ≤ y ≤ 1; 

0,

fora.

Problema 260 La variable aleat` oria Y te per distribuci´ o una Exp(1). Definim   1, si Y < ln n; Xn =  0, altrament.

Obtenir la funci´ o de distribuci´ o de Xn i estudiar la converg`encia en llei de les Xn . Problema 261 La probabilitat de que un virus inform` atic infecte el nostre ordinador ´es 0, 1. Si l’ordinador est` a infectat, un sistema antivirus ho detecta amb probabilitat pI|D = 0, 95, mentre que en caso de no ho haja fet el sistema detecta falses infeccions amb probabilitat 0, 03. Interesa que el sistema antivirus tinga un alt valor predictiu={probabilitat de que el ordinador estiga infectat quan l’antivirus ho detecta}. Calcular-lo a partir de les dades anteriors. Si volem augmentar-lo, qu`e ens conv´e, augmentar x o rebaixar y?

5.4. 5.4.1.

8 de junio de 2004 Castellano

Problema 262 La variable aleatoria X, que toma valores en el intervalo [0,2], tiene por densidad la recta que pasa por (2,0) con pendiente negativa. Obtener su funci´ on de densidad f (x) y calcular P (|X − E(X)| ≤ 1/2). ¿Qu´e cota obtendr´ıamos para esta probabilidad si utiliz´ aramos la desigualdad de Chebychev? Problema 263 Supongamos una clase de n estudiantes. Uno de ellos conoce un rumor que cuenta a uno de sus compa˜ neros elegido al azar. A su vez este segundo estudiante vuelve a contarlo a otro compa˜ nero elegido al azar y distinto del que se lo ha contado. Este rumor sigue propag´ andose del mismo modo. En cada ocasi´ on el estudiante lo cuenta a otro elegido al azar entre los n del grupo, excluyendo a aqu´el que se lo cont´ o. ¿Cu´ al es la probabilidad de contar la historia k veces sin que se la cuenten dos veces al mismo individuo? Sugerencia.- Definid los sucesos Ai ={la historia no se le repite de nuevo a alguien que ya la conoce cuando se cuenta por i-´esima vez}, i = 1, . . . , k. A partir de estos sucesos se puede definir el suceso de inter´es. Problema 264 La variable aleatoria X se distribuye exponencialmente con par´ ametro Y , que es a su vez una variable aleatoria uniforme en [1, 4]. Obtener la distribuci´ on conjunta de X e Y y la esperanza y la varianza de X.

5.4. 8 DE JUNIO DE 2004

49

Problema 265 Se redondean 20 n´ umeros al entero m´ as cercano y despu´es se suman. Supongamos que los errores de redondeo son independientes y uniformemente distribuidos en el intervalo [− 12 , 12 ]. Se pide determinar la probabilidad de que la suma obtenida difiera de la suma de los 20 n´ umeros originales en m´ as de 3 unidades. Problema 266 Consideremos el siguiente procedimiento: 1. Generamos U con distribuci´ on uniforme en [0, 1]. 2. Tomamos Y = − λ1 ln(1 − U ) siendo ln el logaritmo neperiano y λ una constante positiva. 3. Tomamos X = [Y ] donde [Y ] es la parte entera por exceso de Y . Se pide: 1. La distribuci´ on de probabilidad de la variable aleatoria Y . 2. La funci´ on de probabilidad de la variable X. Comprobar que tiene una distribuci´ on geom´etrica. Problema 267 Tenemos una urna con 12 bolas numeradas de 1 a 12. Extraemos dos bolas y denotamos por X1 y X2 los valores que observamos en la primera y en la segunda extracci´ on. Sea X la variable definida como el m´ aximo de las dos extracciones. Se pide: 1. La funci´ on de distribuci´ on de la variable X si suponemos que las dos extracciones se realizan con reemplazamiento. 2. La funci´ on de distribuci´ on de la variable X si suponemos que no hay reemplazamiento entre las extracciones sucesivas. 3. La media de la variable X en las dos situaciones anteriores.

Segundo parcial: problemas 3, 4, 5 y 6. Examen final de toda la asignatura: problemas 1, 2, 4 y 5.

5.4.2.

Valenciano

Problema 268 La variable aleat` oria X, definida en l’interval en [0, 2], t´e per densitat la recta que pasa per (2,0) amb pendent negativa. Obtenir f (x) i calcular P (|X − E(X)| ≤ 1/2). ¿Quina cota obtindr´ıem per aquesta probabilitat si empr` arem la desigualtat de Chebychev? Problema 269 A una classe hi ha n estudiants. Un d’ells coneix un rumor que conta a un company triat a l’atzar. Aquest segon estudiant el conta a un tercer, distint del que li’l va contar a ell i triat tamb´e a l’atzar. El rumor continua propagant-se d’aquesta forma: un estudiant el conta a un altre triat a l’atzar i diferent del que li’l va contar a ell. Quina ´es la probabilitat de que en contar-lo k voltes el rumor no haja tornat a cap dels estudiants que ja el coneixien? Sugger` encia.- Definiu els esdeveniments Ai ={el rumor no es repeteix a cap dels que ja el coneixen quan es conta per i-`esima volta}, i = 1, . . . , k. Aquests esdeveniments ajuden a definir l’esdeveniment que ens interessa Problema 270 La variable aleat` oria X t´e una distribuci´ o exponencial amb par` ametre Y , que ´es al seu torn una variable aleat` oria uniforme sobre [1, 4]. Obtenir la distribuci´ o conjunta de X i Y i la esperan¸ca i la varian¸ca de X.

´ CAP´ITULO 5. EXAMENES PREVIOS

50

Problema 271 S’aproximen 20 n´ umeros mitjan¸cant l’enter m´es proper i despr´es se sumem. Suposem que els errors comesos s´ on independents i uniformes en l’interval [− 21 , 12 ]. Volem obtenir la probabilitat de que la suma obtinguda s’allunye de la suma dels 20 n´ umeros originals m´es de 3 unitats. Problema 272 Considerem el seg¨ uent proc´es: 1. Generem U amb distribuci´ o uniforme en [0, 1]. 2. Definim Y = − λ1 ln(1 − U ), on ln ´es el logaritme neperi` a i λ una constant positiva. 3. Prenim X = [Y ], la part entera per exc´es d’[Y ]. Es demana: 1. La distribuci´ o de probabilitat de la variable aleat` oria Y . 2. La funci´ o de probabilitat de la variable X. Comprovar que t´e una distribuci´ o geom`etrica. Problema 273 En una urna hi han 12 boles numerades de l’1 al 12. En tirem fora dues i designem per X1 i X2 els valors observats a la primera i a la segona extracci´ o, i per X el m` axim de totes deus extraccions. Es demana: 1. La funci´ o de distribuci´ o de la variable X si les dues extraccions s´ on fetes amb reempla¸cament. 2. La funci´ o de distribuci´ o de la variable X si les dues extraccions s´ on fetes sense reempla¸cament. 3. La mitjana de la variable X en tots dos casos.

Segon parcial: problemes 3, 4, 5 i 6. Examen final de tota la mat` eria: problemes 1, 2, 4 i 5.

5.5. 5.5.1.

9 de febrero de 2005 Castellano

Problema 274 ¿Cu´ al es la probabilidad de que una mano de p´ oquer contenga s´ olo una pareja? Nota: una baraja de p´ oquer tiene cuatro palos y de cada palo hay 13 cartas. En una mano se sirven cinco cartas. Problema 275 Una urna contiene n papeletas numeradas de 1 a n inclusive. Extraemos r al azar. Sea X el n´ umero mayor obtenido si las papeletas se reemplazan despu´es de cada extracci´ on y sea Y el n´ umero mayor si las papeletas no se reemplazan en la urna. Determinar las funciones de distribuci´ on, las funciones de cuant´ıa (o probabilidad) y demostrar que FY (k) < FX (k) para 0 < k < n.

(5.1)

Problema 276 Calcular la probabilidad de poder formar un tri´ angulo con dos puntos elegidos en el intervalo [0, 1] seg´ un el m´etodo siguiente: Elegimos un punto al azar y a continuaci´ on uno de los trozos, elegido al azar, lo dividimos en dos partes iguales. Calcular, para este m´etodo, la probabilidad de que el tri´ angulo sea obtus´ angulo. Nota: dado un tri´ angulo cuyos lados miden a, b y c y cuyos a ´ngulos opuestos son A, B y C, respectivamente, se verifica que c2 = a2 + b2 − 2ab cos C.

51

5.6. 21 DE JUNIO DE 2005

Problema 277 Probar que para cualquier funci´ on de densidad de probabilidad se verifica Z +∞ 1 f (z)dz = 0. l´ım x x→+∞ z x

5.5.2.

Valenciano

Problema 278 Quina ´es la probabilitat de que una m` a de p` oquer tinga sols una parella? Nota: una baralla de p` oquer t´e quatre pals i de cada pal hi han 13 cartes. En una m` a s’en serveixen 5. Problema 279 Una urna cont´e n paperetes numerades de 1 a n tots dos inclosos. En traiem r a l’atzar. Siga X el major n´ umero tret si les extraccions han estat fetes amb reempla¸cament, i siga Y el major n´ umero quan les extraccions han estat fetes sense reempla¸cament. Determinar les funcions de distribuci´ o, les funcions de quantia (o probabilitat) i demostrar que FY (k) < FX (k) per a 0 < k < n.

(5.2)

Problema 280 Calcular la probabilitat de poder formar un triangle amb els tres segments obtinguts en triar dos punts a l’interval [0, 1] d’acord amb el seg¨ uent m`etode: triem un punt a l’atzar i tot seguit un dels trossos, tamb´e triat a l’atzar, el dividim en dos parts iguals. Calcular tamb´e la probabilitat de que el triangle siga obtusangle. Nota: per a un triangle de costats a, b i c amb angles oposats A, B i C respectivament es verifica que c2 = a2 + b2 − 2ab cos C. Problema 281 Probar que per a qualsevol funci´ o de densitat de probabilitat es cert que Z +∞ 1 f (z)dz = 0. l´ım x x→+∞ z x

5.6. 5.6.1.

21 de junio de 2005 Castellano

Problema 282 En una urna hay una bola roja. Extraemos tres cartas de una baraja francesa(52 cartas repartidas en 4 palos) y a˜ nadimos a la urna tantas bolas verdes como ases hayamos extra´ıdo. A continuaci´ on lanzamos 2 veces una moneda cuya probabilidad de cara es p = 1/5 y a˜ nadimos tantas bolas rojas como cruces hayamos obtenido. Finalmente llevamos a cabo 2 extracciones con reemplazamiento de la urna. Si X es el n´ umero de bolas verdes a˜ nadidas a la urna e Y el n´ umero de bolas rojas a˜ nadidas a la urna, 1. Obtener la funci´ on de probabilidad de X. 2. Obtener la funci´ on de probabilidad de Y . 3. Si las dos bolas extra´ıdas con reemplazamiento son rojas, ¿cu´ al es la probabilidad de no haber obtenido ning´ un as al extraer las 3 cartas de la baraja francesa? Problema 283 Sea T ∼ U (−1/c, 1/c), c > 0, y definamos Y = cT . Para k ∈ N , las variables aleatorias Xk se definen mediante,   −1, si −1 < Y < −1/k; 0, si −1/k ≤ Y < 1/k; Xk =  1, si 1/k ≤ Y < 1.

´ CAP´ITULO 5. EXAMENES PREVIOS

52

Demostrar que Xk converge en ley a la variable aleatoria X con funci´ on de probabilidad fX (−1) = fX (1) = 1/2 y fX (x) = 0, x ∈ / {−1, 1}. Problema 284 Definamos la funci´ on f (x) =



ax−(s+1) , si x > r; 0, si x ≤ r,

con r, s > 0. 1. Determinar a para que f (x) sea una funci´ on de densidad de probabilidad. 2. Si la variable aleatoria X tiene por densidad f (x), ¿para qu`e valores de s existir´ a su esperanza? Problema 285 Probar utilizando el Teorema Central del L´ımite que e−n

n X nk n 1 −→ . k! 2 k=0

Problema 286 Sobre un c´ırculo cuyo radio R es aleatorio con funci´ on de densidad  2 r   , r ∈ [0, 3]; 9 fR (r) =   0 en el resto,

elegimos un punto al azar. Si X designa la distancia del punto al origen, obtener 1. La funci´ on de distribuci´ on y la funci´ on de densidad de X|R = r. 2. La media y la varianza de X.

Problema 287 Un autob´ us tiene en su recorrido 15 paradas. Supongamos que en la primera parada suben 20 personas. Cada una de ellas elige al azar e independientemente de las otras en cu´ al de las 14 paradas restantes quiere bajar. 1. Si Xi es una variable aleatoria que vale 1 si alguna de las personas baja en la parada i y 0 en caso contrario, calcular su distribuci´ on de probabilidad. 2. Calcular el n´ umero medio de paradas que debe realizar el autob´ us para que bajen todos los pasajeros.

Toda la materia: P1, P2, P3 y P6 Segundo parcial: P2, P4, P5 y P6 5.6.2.

Valenciano

Problema 288 Tenim una urna amb una bola roja. Triem a l’atzar 3 cartes d’una baralla francesa (52 cartes repartides en 4 pals) i afegim a la urna tantes boles verdes com asos hem tret. Despr´es llancem 2 vegades una moneda que t´e una probabilitat p = 1/5 de traure cara i afegim a la urna tantes boles roges com creus hem obtingut. Finalment fem 2 extraccions amb reempla¸cament de la urna. Si X i Y representen el n´ umeros de boles verdes i roges, respectivamente, que hem afegit a la urna,

53

5.6. 21 DE JUNIO DE 2005

1. Obtenir la funci´ o de probabilitat d’X. 2. Obtenir la funci´ o de probabilitat d’Y . 3. Si las dos boles extretes amb reempla¸cament son roges, quina ´es la probabilitat de no haber obtingut cap as al triar les 3 cartes de la baralla francesa? Problema 289 Siga T ∼ U (−1/c, 1/c), c > 0. Definim Y = cT i per a k ∈ N definim la variable Xk mitjan¸cant,   −1, si −1 < Y < −1/k; 0, si −1/k ≤ Y < 1/k; Xk =  1, si 1/k ≤ Y < 1. Demostrar que Xk convergeix en llei a la variable aleat` oria X amb funci´ o de probabilitat fX (−1) = fX (1) = 1/2 i fX (x) = 0, x ∈ / {−1, 1}. Problema 290 Definim la funci´ o f (x) =



ax−(s+1) , si x > r; 0, si x ≤ r,

amb r, s > 0. 1. Determineu a per a que f (x) siga una funci´ o de densitat de probabilitat. 2. Si la variable aleat` oria X t´e per densitat f (x), per a quins valors d’ s existir` a la seua esperan¸ca? Problema 291 Emprant el Teorema Central del L´ımit proveu que e−n

n X nk

k=0

k!

n

−→

1 . 2

Problema 292 Sobre un cercle de radi R, aleatori amb funci´ o de densitat  2 r   , r ∈ [0, 3]; 9 fR (r) =   0 fora,

triem un punt a l’atzar. Si per X designem la dist` ancia del punt a l’origen, obtenir 1. La funci´ o de distribuci´ o i la funci´ o de densitat de X|R = r. 2. La mitjana i la vari` ancia d’X. Problema 293 Un autob´ us t´e 15 parades al llarg del seu recorregut. Suposem que en la primera parada pugen a l’autob´ us 20 persones. Cadascuna d’elles tria a l’atzar i independentment de les altres en quina de les altres 14 parades vol baixar. 1. Si Xi ´es una variable aleat` oria que val 1 si alguna de las persons baixa en la parada i i 0 en cas contrari, calcular la seua distribuci´ o de probabilitat. 2. Calcular la mitjana del n´ umero de parades que deu fer l’autob´ us per a que baixen tots els passatgers.

Tota la mat` eria: P1, P2, P3 i P6 Segon parcial: P2, P4, P5 i P6

´ CAP´ITULO 5. EXAMENES PREVIOS

54

5.7. 5.7.1.

6 de junio de 2006 Castellano

Problema 294 Cuando cierta m´ aquina est´ a bien ajustada el 60 % de las piezas que produce son de calidad alta, el 30 % son de calidad media y el resto de calidad baja. Cuando se desajusta, en cambio, el 40 % de las piezas que produce son de calidad alta, el 40 % son de calidad media y el resto de calidad baja. 1. Supongamos que en un momento dado la m´ aquina est´ a bien ajustada, ¿cu´ al es la probabilidad de que si se eligen 6 piezas al azar se obtengan 3 de buena calidad y 2 de calidad media? 2. Supongamos ahora que la m´ aquina est´ a desajustada, ¿cu´ al es la probabilidad de que si se eligen 6 piezas al azar se obtengan 2 piezas de cada tipo? 3. Se sabe que esta m´ aquina est´ a bien ajustada el 80 % del tiempo pero en este momento los responsables no saben si lo est´ a o no. Un estudiante de c´ alculo de probabilidades propone que se elijan al azar 5 piezas y que se cuenten cuantas hay de cada tipo. En funci´ on del resultado se puede saber en qu´e estado se encuentra la m´ aquina. Los responsables de la m´ aquina aceptan su sugerencia y observan 2 piezas de calidad alta, 2 de calidad media y una de calidad baja. ¿En qu´e estado es m´ as veros´ımil que se encuentre la m´ aquina? Problema 295 1. Estudia la convergencia en ley de {Xn }n≥1 siendo Xn una variable aleatoria uniformemente distribuida en { n1 , n2 , . . . , n−1 n , 1}, para n ≥ 1 2. Comprueba que {Yn }n≥2 converge en probabilidad a una constante, siendo Yn = Xn ∼ U (0, 1) para n ≥ 2.

Xn log n ,

Problema 296 Un jugador juega una cantidad inicial de dinero 1e. En cada partida, con igual probabilidad, duplica su dinero o lo reduce a la mitad. Despu´es de jugar n partidas ¿cu´ al es la ganancia esperada del jugador? Problema 297 Consideramos X una variable aleatoria con distribuci´ on uniforme en el intervalo [0, π]. Determinar la funci´ on de densidad de la variable Y = sen(X). Problema 298 Sean X, U y W variables aleatorias independientes con distribuci´ on uniforme en [0, 1]. Definimos Y = XU +(1−X)W . Determinar EY y var(Y ). ¿Tiene Y una distribuci´ on uniforme en [0, 1]? Problema 299 Se ofrece un premio a cualquier persona que cuando lance 600 veces un dado obtenga un m´ınimo de 125 veces el n´ umero 6. Lo intentan 300 individuos. ¿Cu´ al es la probabilidad de que al menos dos de ellos ganen el premio?

Segundo parcial: problemas 2, 3 5 y 6. Examen final de toda la materia: problemas 1, 2, 4 y 5.

5.7. 6 DE JUNIO DE 2006

5.7.2.

55

Valenciano

Problema 300 Una m´ aquina ben ajustada produeix un 60 % de peces de qualitat alta, un 30 % de qualitat mitjana i un 10 % de qualitat baixa. Si es desajusta, les proporcions s´ on 40, 40 i 20, respectivament. 1. Si la m` aquina est` a ben ajustada, quina ´es la probabilitat de que al triar a l’atzar 6 peces 3 siguen d’alta qualitat i 2 de mitjana? 2. Si la m` aquina est` a desajustada, quina ´es la probabilitat de que al triar a l’atzar 6 peces n’hi hagen 2 de cada tipus? 3. En un moment donat no sabem si la m` aquina est` a o no ajustada, tot i que el 80 % del temps ho est` a. Per a saber-ho alg´ u ens proposa triar 5 peces a l’atzar i veure quantes n’hi han de cada tipus i decidir en funci´ o del resultat. Feta la prova trobem 2 peces d’alta qualitat i 2 de mitjana. En quin ´es m´es veros´ımil que es trobe la m` aquina? Problema 301 1. Estudia la converg`encia en llei de {Xn }n≥1 on Xn una variable aleat` oria , 1}, n ≥ 1 uniformemente distribu¨ıda en { n1 , n2 , . . . , n−1 n 2. Comprova que {Yn }n≥2 convergeix en probabilitat a una constant, essent Yn = Xn ∼ U (0, 1) n ≥ 2.

Xn log n ,

Problema 302 Un jugador juga una quantitat inicial c. En cada partida, amb igual probabilitat, la quantitat jugada es duplica o es redueix a la meitat. Despr´es de jugar n partides, quin ´es el guany esperat del jugador? Problema 303 Considerem X una variable aleat` oria amb distribuci´ o uniforme en l’interval [0, π]. Determinar la funci´ o de densitat de la variable Y = sin(X). Problema 304 Les variables aleat` ories X, U i W s´ on independents i totes tres U (0, 1). Definim Y = XU + (1 − X)W . Determinar E(Y ) i var(Y ). T´e Y una distribuci´ o U (0, 1)? Problema 305 S’ofereix un premi a qualsevol que en 600 llan¸caments d’un dau obtinga un m´ınim de 125 sisos. Ho intentan 300 individus. Quina ´es la probabilitat de que al menys dos de’ells guanyen el premi?

Segon parcial: problemes 2, 3 5 i 6. Examen final de tota la mat` eria: problemes 1, 2, 4 i 5.

View more...

Comments

Copyright ©2017 KUPDF Inc.
SUPPORT KUPDF